Acute SCE exam full mark

¡Supera tus tareas y exámenes ahora con Quizwiz!

CNS and Eye

CNS and Eye

GIT and Liver

GIT and Liver

Respiration

Respiration

Question 2 of 31 A 59-year-old lady presents with a very unsteady gait and difficulty in weight bearing which has left her non-ambulatory. The condition has progressed quickly over the past four days following an episode of gastroenteritis. On further questioning the symptoms are consistent with an ascending neuropathy in the lower limbs associated with paraesthesia. A lumbar puncture shows raised protein in the cerebrospinal fluid (CSF). You commence her on treatment and ask for her forced vital capacity (FVC) to be monitored. Intubation is indicated when the FVC reading falls below which reading? (Please select 1 option) <20% of FVC predicted <25% of FVC predicted <30% of FVC predicted <40% of FVC predicted <50% of FVC predicted

<30% of FVC predicted This is the correct answer Due to the nature of the disease patients with Guillain-Barré (GBS) are at high risk of respiratory failure. Patients who remain spontaneously breathing, require very close monitoring to determine need for ventilator support. The most commonly used parameter to monitor for respiratory failure is regular vital capacity measurements. It is important to set clear guidance for staff recording the measurements so that a case of respiratory decline is acted upon in appropriately. Alongside this clinical parameters such as oxygen saturations, respiratory rate and use of accessory muscles as well arterial blood gas testing (showing a rise in PaCO2), may be used to check on respiratory function. Intensivists will want to be informed about a GBS patient early so get them involved and make them aware. Approximately 30% of patients with GBS require intubation. Reference: Henderson RD, Lawn ND, Fletcher DD, et al. The morbidity of Guillain-Barré syndrome admitted to the intensive care unit. Neurology. 2003;60:17-21. Pearse RM, Draper A, Grounds RM. Non-invasive ventilation to avoid tracheal intubation in a patient with Guillain-Barré syndrome. Br J Anaesth. 2003;91:913-6. van Der Meché FG, van Doorn PA. Guillain-Barré Syndrome. Curr Treat Options Neurol. 2000;2:507-516.

There is a widespread public health campaign to try to stop smoking. Doctors have a key responsibility to impart advice in this area. Which of the following smoking cessation aids are not currently available on the NHS? (Please select 1 option) Champix (varenicline) Electronic cigarettes containing nicotine Gum containing nicotine Inhalant devices containing nicotine Zyban (bupropion)

A 2013 paper in the Lancet1 suggested electronic cigarettes may be as effective as patches, although the numbers recruited for the study were too small for the findings to be statistically significant. There were no short term (six month) adverse effects. e-Cigarettes do contain chemicals other than nicotine, including diethylene glycol. However these chemicals are present in much greater quantities in tobacco cigarettes. It is likely that e-cigarettes are less harmful than traditional tobacco cigarettes but until their long term safety and efficacy is proven they are unlikely to be NICE approved or available on the NHS. Reference: Bullen C, Howe C, Laugesen M, et al. Electronic cigarettes for smoking cessation: a randomised controlled trial. Lancet. 2013;382:1629-37.

A 78-year-old man presents with episodes of syncope. His 12 lead ECG is as follows: Which of the following is the correct diagnosis? (Please select 1 option) Atrial fibrillation with complete heart block Atrial fibrillation with left bundle branch block Complete heart block Hypertropic cardiomyopathy Paced rhythm

Atrial fibrillation with complete heart block can easily be missed. Patients presenting in this rhythm require permanent pacemaker insertion. Key features when interpreting this ECG include: lack of P waves regular ventricular rhythm, and widened QRS complexes (indicating ventricular escape rhythm). Atrial fibrillation with left bundle branch block is incorrect as the ventricular activity is regular on this ECG. In AF and LBBB one would expect irregularly irregular QRS complexes. Complete heart block is incorrect due the absence of p waves. Hypertropic cardiomyopathy is incorrect as there are no deep, narrow Q waves and no clear ST/T wave abnormalities (for example, giant T wave inversions in the precordial leads). HCM also often presents with left atrial enlargement on the ECG. Paced rhythm is incorrect due to the absence of regular pacing spikes.

Question 20 of 31 A 23-year-old male of Mediterranean origin presents with an acute red eye which is sensitive to light. You notice what looks like a white fluid level in this eye. He denies any recent eye surgery. On further questioning he explains he gets lots of mouth ulcers. You also notice a rash on his shins which he says is quite painful. What is your most likely diagnosis for this gentleman? (Please select 1 option) Behçet's disease Sarcoidosis Syphilis Tuberculosis Vogt-Koyanagi-Harada disease

Behçet's disease This is the correct answer The symptoms and signs are consistent with that of a likely diagnosis of Behçet's disease. Patients are often from the Silk Route countries, Mediterranean, Middle East and up to Japan. International criteria for the diagnosis of Behçet's are the presence of mouth ulcers and two other manifestations, which include: ophthalmic signs, characteristically with anterior uveitis with a mobile hypopyon, and dermatological signs such as acneform lesions or erythema nodosum, positive pathergy test and genital ulcers. Sarcoidosis is unlikely to present with a uveitis with a hypyon or with mouth ulcers. Syphilis is characterised by a maculopapular rash which is usually on the hands or soles of feet as opposed to shins. The skin manifestations of Vogt-Koyanagi-Harada disease are vitiligo and poliosis (white eyelashes). Also, patients may have hearing and neurological manifestations.

Question 1 of 31 Core Questions A 77-year-old female complains that one half of her vision has disappeared. You confirm this by screening the visual field by confrontation. Upon ophthalmoscopy you notice the following disc appearance: What is the likely cause of this lady's visual field loss? (Please select 1 option) Anterior ischaemic optic neuropathy Arteritic anterior ischaemic optic neuropathy (GCA) Branch retinal artery occlusion Branch retinal vein occlusion Optic nerve drusen

Branch retinal artery occlusion This is the correct answer The image shows a calcific emboli occluding the superior temporal retinal artery and hence giving rise to her hemi-field defect. The retina adjacent to the emboli appears swollen and pale. Remember to carefully examine the disc and in particular both the arteries and veins to look for relevant pathology. A good systemic method is the three Cs and V: Colour Cup Contours, and Vessels. Anterior ischaemic optic neuropathy (AION) can account for the hemi-field defect, however the disc margin would be blurred in the corresponding area of the disc. Arteritic AION would give rise to complete field loss and a swollen unilateral disc. Branch retinal vein occlusion would give rise to multiple flame haemorrhages on one half of the optic disc. Optic nerve drusen would give an appearance of an elevated disc often confused as papilloedema.

A 36-year-old female patient presents to the outpatient clinic with a history of intermittent palpitations, which she describes as being fast and irregularly irregular. Her 12 lead ECG is shown below: Which of the following drugs would be appropriate for use? (Please select 1 option) Adenosine Amiodarone Diltiazem Nebivolol Verapamil

The ECG shows the typical features of a Wolff-Parkinson-White syndrome: a shortened PR interval a slurring and slow rise of the initial upstroke of the QRS complex (delta wave) a widened QRS complex (total duration >0.12 seconds), and ST segment-T wave changes, generally directed opposite the major delta wave and QRS complex. WPW is a pre-excitation syndrome due to an accessory electrical conduction pathway (bundle of Kent), between atria and ventricles. A case of type A pre-excitation, which has left atrioventricular connections, is denoted by a positive R wave will be seen in V1 ('positive delta') on the precordial leads of the electrocardiogram. In type B pre-excitation, which has right atrioventricular connections, there is a predominantly negative delta wave lead V1 ('negative delta'). AV node blockers should be avoided in atrial fibrillation and atrial flutter with WPW or history of it; this includes: adenosine diltiazem verapamil other calcium channel blockers, and beta blockers. In particular adenosine activates the IK, Ach channel in atrial tissue. Activation of the IK, Ach channel shortens the action potential duration, thereby shortening the refractoriness of the atrial tissue and favoring 1:1 atrial to ventricle conduction through the pre-excitation pathway. This potentially leads to unstable ventricular arrhythmias. The definitive treatment of WPW is a destruction of the abnormal electrical pathway by radiofrequency catheter ablation.

A 51-year-old man presents to you with worsening shortness of breath over several years. He is now limited to walking on the flat. His past medical history is unremarkable apart from a history of a 'severe chest infection' aged 21 for which he required intravenous antibiotics as an inpatient. Exploring his exercise tolerance further he tells you that he can walk slowly for approximately 150 yards 'on the flat' before he has to stop. He cannot manage stairs and inclines. He can still drive a car and works as a supervisor on a building site. His job typically does not involve heavy lifting or manual labour. What is his Medical Research Council dyspnoea (MRC) score? (Please select 1 option) MRC 1 MRC 2 MRC 3 MRC 4 MRC 5

The MRC scores are defined as: Grade 1 - "I only get breathless with strenuous exercise" Grade 2 - "I get short of breath when hurrying on the level or up a slight hill" Grade 3 - "I walk slower than people of the same age on the level because of breathlessness or have to stop for breath when walking at my own pace on the level" Grade 4 - "I stop for breath after walking 100 yards or after a few minutes on the level" Grade 5 - "I am too breathless to leave the house". By this definition he would be Grade 3, in that he can walk more than 100 yards but is unable to manage inclines. Reference: Fletcher CM, Elmes PC, Fairbairn AS, Wood CH. The significance of respiratory symptoms and the diagnosis of chronic bronchitis in a working population. Br Med J. 1959;2:257-66.

Which of the following pathologies is not eligible for compensation in England? (Please select 1 option) Asbestos related lung cancer Asbestos related pleural plaques Asbestos related pleural thickening Asbestosis Mesothelioma

The degree of compensation offered by either the employer, previous employer or Department for Work and Pensions will vary depending on the illness and degree of disability, as well as physiological impairment on lung function tests, pain suffered, and loss of earnings experienced due to having the illness. Pleural plaques indicate exposure to asbestos but are benign in nature so are no longer compensable. However, individuals with pleural plaques from asbestos exposure should still be advised to look out for symptoms of malignancy in future such as chest pain, weight loss and haemoptysis, as their previous exposure means they still have a risk of developing an asbestos related lung disease.

Question 1 of 22 Core Questions A 38-year-old man is admitted to the Emergency department with severe lethargy, nausea, vomiting and jaundice. He is normally fit and well but went out mushroom picking in the woods a few days earlier, using a book he received as a birthday present to identify mushrooms. On examination he looks unwell, he has jaundiced sclera and extensive bruising over his arms, legs and abdomen. He is tender in the right upper quadrant. Investigations show: Hb 104 g/L (130-180) WCC 11.5 ×109/L (4-10) PLT 155 ×109/L (130-400) Na 137 mmol/L (135-145) K 4.9 mmol/L (3.5-5.5) Cr 139 µmol/L (70-120) Urea 9.1 mmol/L (2.5-8) AST 2990 U/L (0-35) Bilirubin 82 pmol/L (<26) Albumin 33 g/L (35-50) INR 4.2 - Which is the most likely cause of his presentation? (Please select 1 option) Amatoxin poisoning Muscarine poisoning Norleucine poisoning Orellanine poisoning Psilocybin poisoning

Amatoxin poisoning This is the correct answer Amatoxin poisoning is recognised to lead to presentation with hepatorenal syndrome around three days after ingestion of mushrooms. Amanita phalloides is the best known mushroom containing amatoxin, and may be confused with edible field mushrooms. Interventions include activated charcoal, administration of which is repeated, N-acetylcysteine, and milk thistle. Muscarine poisoning is characterised by increased salivation, perspiration, and lacrimation within 30 minutes of mushroom ingestion. Norleucine poisoning is associated with transient liver impairment, although renal failure associated with norleucine exposure is progressive. Orellanine poisoning is associated with renal failure, and psilocybin poisoning is associated with hallucinations and neurological symptoms. If mushroom poisoning is suspected, it is often helpful for patients to bring in a sample of what they have eaten. Atlases can be used to identify the causative agent. Further Reading: Wild Food UK. Wild Mushroom Guide.

A 66-year-old man with diabetes and a known diagnosis history of post-TB bronchiectasis, presents with a seven day history of cough with green sputum, and fevers. He had been started on amoxicillin by his GP with minimal benefit. On the morning of presentation he had two episodes of fresh red haemoptysis, of around 100 ml in volume each time. When seen in the Emergency department he is comfortable, with all bedside observations within normal limits. Chest x ray shows evidence of predominantly upper zone bronchiectatic change, but no obvious evidence of superadded consolidation. Blood tests reveal: Haemoglobin 124 g/L (115-165) White cell count 11.8 ×109/L (4.0-11.0) Platelet count 135 × 109/L (150-400) Serum sodium 137 mmol/L (137-144) Serum potassium 4.1 mmol/L (3.5-4.9) Serum creatinine 95 µmol/L (60-110) Serum C reactive protein 25 mg/L (<10) Arterial blood gases (on room air) reveal: PO2 8.9 kPa (11.3-12.6) PCO2 4.8 kPa (4.7-6.0) pH 7.43 (7.35-7.45) Bicarbonate 24 mmol/L (21-29) After initial stabilisation measures a diagnosis of an infective exacerbation of bronchiectasis is made and an urgent CT thorax is arranged. What is the most appropriate management plan at this stage? (Please select 1 option) Admit for inpatient antibiotics/observation alone Outpatient antibiotics as per sensitivities Refer for bronchial arteriogram +/- embolisation Refer for urgent flexible bronchoscopy Refer for urgent rigid bronchoscopy under GA

Massive haemoptysis is arbitrarily defined as 100-600 ml of blood expectoration in 24 hours and, in ~90% of cases, is caused by a bronchial arterial bleed (that is, systemic pressures). The immediate mortality is quoted up to 85% in studies. Given the risk of asphyxiation and death with relatively small volumes of blood, prompt action is required, including initial resuscitation and stabilisation. Prompt antibiotics should be administered if an infective cause is suspected. There is no role for flexible bronchoscopy in the acute management of massive haemoptysis, as the airway cannot be protected, and often hypoxaemia precludes the procedure. Rigid bronchoscopy can be effective if a bleeding point is visualised, though it carries the associated risk of a general anaesthetic and is often an unfavourable option depending on the patient's ASA status. Bronchial artery embolisation, offered at tertiary centres, has become a well-established technique for massive haemoptysis with immediate success rates of above 90%, and with improvements in super-selective catheterisation techniques, a low rate of embolic complications. The procedure is done awake and supine, and should be considered for haemostasis control in all patients with massive haemoptysis. Reference: Chun JY, Belli AM. Immediate and long-term outcomes of bronchial and non-bronchial systemic artery embolisation for the management of haemoptysis. Eur Radiol. 2010;20(3):558-65.

Others

Others

A 50-year-old female with known breast cancer and pulmonary and cerebral metastases is admitted with a two day history of breathlessness, and a syncopal episode lasting one minute earlier that day. She has no other past medical history of note. On examination she has a GCS of 15/15, blood pressure 112/56 mmHg, heart rate 123 bpm and respiratory rate of 20/minute. There are no signs of upper or lower limb DVT. ECG shows sinus tachycardia with right bundle branch block. Bedside echo shows signs of right heart strain. Cardiac troponin is mildly elevated. Arterial blood gas on air is as follows: PO2 10.4 kPa (11.3-12.6) PCO2 4.0 kPa (4.7-6.0) pH 7.47 (7.35-7.45) Bicarbonate 22 mmol/L (21-29) CTPA showed a large pulmonary embolus straddling the bifurcation of the main pulmonary trunk. What is the best management plan? (Please select 1 option) Heparin followed by early referral to tertiary centre for consideration of catheter embolectomy IV unfractionated heparin and close observation alone IVC filter insertion Subcutaneous low molecular weight heparin and close observation alone Thrombolysis with tissue plasminogen activator

Though currently haemodynamically stable, the patient has a number of adverse features that have been shown to increase 30 day mortality, namely: CT-evidence of saddle PE echocardiographic evidence of right heart strain, and raised cardiac enzymes. There is currently no evidence to support thrombolysis of sub-massive PE, and expert opinion varies. The patient has intracerebral metastases, which is a contraindication to thrombolysis. However, given her episode of collapse and adverse features, along with central location of the PE, she may well benefit from interventional techniques, such as catheter embolectomy, rheolysis (clot dissolution) or localised thrombolysis, and her case should be discussed with a tertiary cardiothoracic centre. Reference: Howard LS. Thrombolytic therapy for submassive pulmonary embolus? PRO viewpoint. Thorax. 2014;69:103-105.

Question 8 of 15 A 43-year-old patient presents to you with bilateral knee and ankle pain. The pain is worse after movement and improves with rest and paracetamol. It is a dull ache in the centre of the joint. There is no history of trauma or recent viral illness. On examination the patient is obese (BMI 40). There is a good passive range of motion in all the joints. You send an auto immune screen and request x rays, however you suspect the obesity may be contributing to the pain and anticipate early changes of osteoarthritis. What percentage of the adult population in the UK are obese? (Please select 1 option) 0-10% 10-20% 20-30% 30-40% 40-50%

20-30% This is the correct answer Obesity is associated with significant health outcomes including: hypertension type 2 diabetes heart disease osteoarthritis some types of cancer (breast, colon) stroke sleep apnoea gastro-oesophageal reflux disease fatty liver, and depression. With obesity becoming a global epidemic, health services need to be aware of the challenge obesity presents, and be prepared economically to be able to manage the complications associated with the condition. Reference: Health & Social Care Information Centre (HSCIC). Statistics on Obesity, Physical Activity and Diet - England, 2013.

Question 3 of 15 During a night shift in the acute medical unit a patient with a history of schizoaffective disorder being treated for a paracetamol overdose tries to self discharge. On his notes you see that the he has a high risk of repeat self harm. You decide to detain him on the ward for the next 72 hours until you can contact the psychiatric team. Under what section of the Mental Health Act are you able to detain this patient in these circumstances? (Please select 1 option) 2 3 5(2) 5(4) 136

5(2) This is the correct answer Clinicians should be aware of the common sections on the Mental Health Act. Section 5(2) relates to doctors' holding power. This should be used for patients who have already been admitted to the hospital (therefore it is not applicable for patients in the emergency department, the outpatients department or for those who have come in under section 136). It lasts for 72 hours and is not renewable. Section 2 covers compulsory admission for assessment and treatment for 28 days. Section 3 covers compulsory admission for treatment for up to six months. An application in terms of sections 2 and 3 can be made by the nearest relative, or an AMHP (approved mental health practitioner). The patient must then be seen by two doctors, one of whom is section 12 approved - this is usually the consultant psychiatrist or psychiatric senior registrar. Under section 5(4) a nurse may detain a patient who is receiving treatment for mental disorder for a period of six hours. This is generally used to prevent a patient from leaving when a doctor is not immediately available to assess whether detaining the patient under section 5(2) is appropriate. Under section 136 a police officer can remove a member of the public from a public place to a place of safety if he thinks it necessary to do so in the interests of that person or for the protection of other persons. The patient can be detained for a period not exceeding 72 hours and needs to be seen by a psychiatrist or suitable registered medical practitioner in this time. Reference: Legislation.gov. Mental Health Act 1983.

Question 5 of 31 Core Questions A 26-year-old lady presents to you with headache, neck stiffness and blurred vision. She has presented with her mother and they are both very concerned. The history is one of gradually increasing severity of headache over two months. Her last travel abroad was to Brazil four months ago for a week on holiday. On examination she is apyrexial, BP 150/90 mmHg, HR 60/regular, her BMI is 38. There is a centrocaecal scotoma in her right eye and peripheral visual field constriction. Fundoscopy reveals bilateral papilloedema. Her gait is a bit unsteady but otherwise there is little to find neurologically. A CT and MR brain are normal and an LP shows raised intracranial pressure, but the CSF analysis is otherwise negative. Blood tests are all normal except for total cholesterol and LDL which are high. Which of the following treatments is most appropriate next step? (Please select 1 option) Acetazolamide po Aciclovir iv Ceftriaxone iv Dexamethasone po Doxycycline po

Acetazolamide po This is the correct answer This is a case of idiopathic intracranial hypertension (IIH) or pseudotumour cerebri (also known as benign intracranial hypertension). Headache and visual symptoms are the most common presentation, though sixth nerve palsy is also associated with this due to raised intracranial pressures. Treatment is weight loss in those who are overweight, lumbar puncture to reduce pressure (though daily lumbar punctures are no longer advocated) and treatment with acetazolamide. If symptoms continue to progress despite these measures, lumboperitoneal shunt may be required.

Question 5 of 22 A 67-year-old Indian lady is referred to you with symptoms of retrosternal burning, epigastric discomfort and dysphagia, predominantly to solids. She is a vegetarian. She describes that symptoms have been progressing over the past few months, but the most recent episode worried her as she regurgitated a completely undigested meal of chapatti and lentils. She is a thin lady and her weight has been stable. Blood tests reveal a mild microcytic anaemia (Hb 111 g/L, MCV 72 fL). Bowel motions are regular. Her chest x ray reveals an air fluid level behind the heart seen. Which of the following is the most likely diagnosis? (Please select 1 option) Achalasia Boerhaave syndrome Hiatus hernia Oesophageal carcinoma Pneumopericardium

Achalasia This is the correct answer This lady has symptoms of retrosternal burning, epigastric discomfort and regurgitation of her food. The diagnosis is achalasia, her anaemia is likely to be related to her diet, though separate investigation is required to determine the cause. A differential diagnosis for achalasia may include gastro-oesophageal reflux disease (GORD) and oesophageal malignancy posing as pseudo-achalasia. In GORD dysphagia to solids and liquids and regurgitation of undigested food would be abnormal. In malignancy, there is likely to be a history of significant weight loss. Diagnosis of achalasia is by using barium swallow or oesophageal manometry. Expected findings on barium swallow include a 'rat tail' appearance as the barium is unable to flow freely through the lower oesophageal sphincter. On manometry there is likely to be aperistalsis of the distal two thirds of the oesophagus. Treatment is with endoscopic dilatation or surgical myotomy.

Question 22 of 22 A 27-year-old man presents to the Emergency department for review with progressive lethargy over the past two to three weeks. He now complains of nausea and is anorexic with jaundiced sclerae. He works as a labourer and has a stable partner and does not abuse drugs. The only history of note involves travel to a stag party overseas two months earlier, during which he received a tattoo. Examination confirms jaundiced sclerae. There are no signs of chronic liver disease. His abdomen is soft and non-tender. Investigations reveal: Hb 138 g/L (135-180) WCC 9.8 ×109/L (4.5-10) PLT 284 ×109/L (150-450) Na 138 mmol/L (135-145) K 5.0 mmol/L (3.5-5.5) Cr 112 µmol/L (70-110) AST 580 IU/L (5-35) Alkaline phosphatse 180 U/L (35-100) Bilirubin 60 µmol/L (<26) Hep C RNA Positive Hep C antibody Negative HBsAg Negative Anti-HBc anitbody Positive Anti-HBs antibody Positive Which of the following is the most likely diagnosis? (Please select 1 option) Acute hepatitis A Acute hepatitis B Acute hepatitis C Chronic hepatitis B Chronic hepatitis C

Acute hepatitis C This is the correct answer Whilst acute presentation with hepatitis C infection is unusual, it is seen in up to 20-30% of patients with the disease. Modes of presentation can include: severe tiredness and lethargy raised transaminases, and clinical jaundice. The average for time from infection to onset of symptoms is six to seven weeks, and for seroconversion it is eight to nine weeks. This accounts for the presentation with symptoms, and presence of mRNA for hepatitis C, and the fact that antibodies are yet to appear. Exposure to the infection at the tattoo parlour is most likely. The hepatitis B serology seen here implies previous cleared natural infection, with an absence of antigen, coupled with the presence of antibodies to both core and surface antigen. In vaccination, only antibodies to surface antigen proteins are seen. In hepatitis A, a short prodromal GI illness usually pre-dates the onset of jaundice, and we would not expect to find hepatitis C mRNA.

Question 10 of 15 Core Questions A 43-year-old man with a known nut allergy presents to you with a Glasgow coma scale of E3 M3 V3 after a meal at a Chinese restaurant. He has signs of haemodynamic instability and you suspect anaphylaxis. Which is the most appropriate first step? (Please select 1 option) Administer epinephrine 1:1000 500 mcg IV Administer epinephrine 1:1000 500 mcg IM Administer epinephrine 1:1000 500 mg IM Intravenous chlorpheniramine, hydrocortisone and NIV Intubate the airway

Administer epinephrine 1:1000 500 mcg IM This is the correct answer This may seem like a simple test of knowledge, but anaphylaxis is a severe hypersensitivity reaction which can be life threatening. Knowing how to treat it correctly in the emergency setting is essential. Progression in anaphylaxis is rapid and so identification of symptoms using the ABCDE approach and administering treatment early is important. In genuine presentations the patient is likely to present in the peri-arrest state. You should call for back up early. Although intubation may be required, practically administering epinephrine will occur first; there is no mention of facial swelling in the vignette.

Question 5 of 15 Core Questions A 23-year-old microbiology laboratory technician presents with joint pains and a rash. Due to the severity of the pain she is unable to mobilise despite taking regular ibuprofen for the past few days. She is febrile at 39°C and has a rash on her right thigh spreading to her trunk. This is a non-pruritic, maculopapular, and 'salmon pink' in appearance. She is not sexually active but has suffered a recent needle-stick injury and is very concerned that she may have contracted HIV. On examination, her splenic tip is palpable. Bloods tests results are as follows: Hb 120 g/L WCC 15 ×109/L Plts 480 ×109/L Renal function normal Liver function raised AST and ALT Ferritin 30,000 ng/ml CRP 123 mg/L ESR 110 mm/hr HIV pending Which of the following is your diagnosis? (Please select 1 option) Adult onset Still's disease (ASD) Drug reaction HIV seroconversion Rheumatoid arthritis (RA) Varicella zoster

Adult onset Still's disease (ASD) This is the correct answer This is a case of adult onset Still's disease. The defining criteria are the Yamaguchi criteria: Major Minor Fever >39 degrees for 1 week or more Sore throat Salmon pink non-pruritic rash on trunk or extremities Lymphadenopathy Arthralgia for >2 weeks Abnormal liver tests Raised WCC >10 ×109/L Hepato- or splenomegaly The absence of ANA or rheumatoid factor on blood tests and the very raised ferritin are also clues to the diagnosis. Treatment is with aspirin and NSAIDS.

Question 15 of 22 A 33-year-old health care assistant is admitted with a flare of ulcerative colitis. He has a known diagnosis of left sided disease. He now presents with bloody stools >6 times per day. He has lost 3 kg in weight and is complaining of severe crampy abdominal pain and tenesmus. Observations show a HR of 93/min, BP 105/60 mmHg, temperature 37.8°C, saturations 97% air. His normal treatment regime includes mesalazine 2.4 g/day and mebeverine PRN. Which of the following would be appropriate? (Please select 1 option) Increase mesalazine to 4.8 g/day (max dose) Intravenous fluids and antibiotics Septic screen and oral glucocorticoids Steroid enema and 5-aminosalicylic suppositories All of the above

All of the above Correct Severe ulcerative colitis should be managed aggressively and treated with combination therapy. Oral glucocorticoids and high dose oral 5-aminosalicylic acid (5-ASA) (for example, mesalamine 4.8 g/day), 5-ASA or steroid suppository, and 5-ASA, steroid enema or foam can be used. Patients with signs of systemic toxicity should also receive antibiotics, though initiation of oral glucocorticoids should not be delayed until the results of stool studies and cultures are available. This is a case of severe ulcerative colitis. Some of the markers of severity include: stools >6 per day haemoglobin <105 g/L raised inflammatory markers temperature >37.5°C HR >90/min, and weight loss. Treatment is combination therapy, enteral nutrition (elemental diet), thromboprophylaxis and early review by gastroenterology. An abdominal x ray to exclude toxic megacolon should also be conducted. Anticholinergics and anti-diarrhoeals are best avoided in the acute phase. Patients who fail to respond to the above measures after 7-10 days are considered to be refractory cases, and may require treatment with ciclosporin and anti-TNF therapy. Reference: Mowat C, Cole A, Windsor A, et al. Guidelines for the management of inflammatory bowel disease in adults. Gut. 2011;60:571-607.

A 78-year-old man who is normally fit and well presents to you after 'feeling dizzy' whilst waiting for a bus. He denies a history of chest pain or palpitation. He remembers the episode clearly and there is no history of syncope. On examination the patient is oriented, his heart rate is 38/min and blood pressure 140/50 mmHg, apyrexial, sats 98% air. Heart sounds are normal with no added sounds. Lungs are clear on auscultation with no crackles. Blood tests reveal the following: WCC 9 ×109/L Hb 130 g/L Plts 387 ×109/L Na 135 mmol/L K 4.3 mmol/L Urea 14 mmol/L Creatinine 187 µmol/L ECG is shown below: What would be the most appropriate way to manage him in the immediate setting? (Please select 1 option) Atropine External pacing Observation Percussion pacing Temporary pacing wire

Although this man is haemodynamically stable with no symptoms of chest pain, heart failure or syncope, he is developing acute kidney injury indicating circulatory compromise. He requires pacing. Not all patients with complete heart block require immediate pacing. Signs to look out for include: Asystole >3 seconds Bradycardia <40 /min Reduced conscious level or syncope BP <100 mmHg systolic Rising creatinine Urine output <30ml/hr Secondary ventricular arrhythmias.

Question 8 of 31 A 43-year-old teacher presents with a new onset of tiredness and lethargy. She has noticed her eyelids drooping towards the end of the day and has even experienced double vision. She confirms that her symptoms improve again with rest. You suspect a diagnosis of myasthenia gravis but she has tested negative for acetylcholine receptor antibodies. Which other antibody could you test for in this lady? (Please select 1 option) Anti-CCP Anti-Hu antibodies Anti-muSK antibodies Anti-NMDA receptor antibodies Voltage gated calcium channels

Anti-muSK antibodies This is the correct answer Up to 15% of patients with myasthenia gravis may test negatively for acetylcholine receptor antibodies. Myasthenia gravis is a difficult diagnosis to make. It is more common in those with a family history of autoimmune disorders, and symptoms may be precipitated in those treated with penicillamine. Acetylcholine receptor antibodies are present in up to 85% patients. Other tests to help confirm the diagnosis include the edrophonium test (not routinely used) and nerve conduction studies. Anti-NMDA (n-methyl d-aspartate) receptor antibodies are seen in autoimmune encephalitis. Voltage gated calcium channels are associated with Lambert-Eaton syndrome. Anti-CCP (cyclic citrullinated peptide) is seen in rheumatoid arthritis. Anti-Hu antibodies are associated with paraneoplastic lesions.

Question 27 of 31 A 75-year-old woman presents with transient monocular blindness to her left eye. Her ABCD2 score is 3. You have arranged for her to be seen at your department's one stop TIA clinic. What would be your management prior to her attending her appointment? (Please select 1 option) Anticoagulation therapy Antiplatelet and statin therapy Antiplatelet therapy only Carotid Doppler scan Do nothing and wait for her clinic appointment

Antiplatelet and statin therapy This is the correct answer Evidence suggests use of both antiplatelet therapy in the form of aspirin and statins are associated with reduced incidence of stroke. Naylor et al.1 write that, "In the EXPRESS study, TIA/TMB patients started statin/antiplatelet therapy in the TIA clinic. [...] The simple combination of being seen early after the index event and starting statin/antiplatelet therapy in the clinic was associated with an 80% reduction in the 90-day risk of stroke along with significant reductions in fatal strokes, readmissions, bed stay and hospital costs." Most hospital protocols will suggest commencing on some oral therapy prior to attendance. A carotid Doppler scan would be arranged within the TIA clinic. Reference: Naylor AR, Robinson TG, Eveson D, Burns J. An audit of management practices in patients with suspected temporary monocular blindness. Br J Ophthalmol. 2013 [Epub ahead of print].

Question 1 of 15 A 34-year-old male cannabis abuser has come over from the psychiatric ward with a one week history of dyspnoea, cough and generally feeling weak. On examination he is tachycardic with a temperature of 38.0°C and on auscultation creps can be heard over both bases of the lungs. On the drug chart you see that he had been titrated with clozapine over the last two weeks for drug resistant schizophrenia. The bloods show a WCC of 5.2 ×109/L with a raised eosinophil count and a C reactive protein (CRP) of 25 mg/L. What is your next step in the management? (Please select 1 option) Arrange for an ECG and an echo Bloods for clozapine level CTPA Gain IV access and administer broad spectrum antibiotics Urine drug screen

Arrange for an ECG and an echo This is the correct answer Clozapine can cause myocarditis, especially in the first stages of commencing treatment. Non-specific signs of pyrexia and tachycardia, along with a raised eosinophil count and CRP should raise suspicion of myocarditis. Discontinuation of clozapine in such cases is essential. The two other serious side effects of clozapine include agranulocytosis (seen in approximately 0.8%) and increased risk of pulmonary embolism. Other side effects include: weight gain insulin resistance lowered seizure threshold excessive salivation urinary incontinence, and over sedation. Despite this it is still a popular choice of treatment amongst psychiatrists as it is regarded as one of the most effective anti-psychotics. Further Reading: UpToDate. Guidelines for prescribing clozapine in schizophrenia.

Question 9 of 31 A patient presents with vertigo. You conduct the Epley manoeuvre and the patient improves. Which of the following is the most likely diagnosis? (Please select 1 option) Benign paroxysmal positional vertigo Cerebellar stroke Cerebellar tumour Meniere's disease Vestibular neuritis

Benign paroxysmal positional vertigo This is the correct answer The Epley manoeuvre is a simple bedside cure for benign paroxysmal positional vertigo (BPPV). It involves moving debris (calcium carbonate) which may have settled in the bottom of the posterior semicircular canal. BPPV is a common presentation. It is thought that debris which drops off from the labyrinth brushes along hairs in the semicircular canal and sends erratic messages down the vestibular nerve resulting in vertigo. A Hallpike manoeuvre by the bedside can confirm the diagnosis and can helpfully negate the need for unnecessary brain radiation. Once the diagnosis is confirmed the Epley manoeuvre can be applied, which will move debris from the affected ear and improve symptoms.

Question 9 of 15 Core Questions A 48-year-old lady presents acutely over the weekend with a flare of her psoriasis. She complains of increasingly severe pain in her small joints, particularly her thumb. She is normally well controlled on methotrexate 25 mg weekly. In the past she has received ciclosporin and acetretin. You suspect diagnosis of psoriatic arthritis, for which you commence paracetamol and a NSAID. What further treatment would be suitable in the longer term? (Please select 1 option) Biologic therapy Increase the dose of methotrexate Light therapy (for example, PUVA) Physiotherapy Steroid therapy

Biologic therapy This is the correct answer This patient's psoriasis has progressed over time and she is now presenting with symptoms of psoriatic arthritis, which may occur in 25-33% of psoriasis patients. She is already on a conventional systemic treatment, methotrexate, which is at the maximum treatment dose. The next line management includes a referral to a specialist for confirmation of the diagnosis and a biologic therapy, for example, TNF alfa inhibitors. These drugs are now widely used to treat inflammatory arthropathies and psoriasis. The acute physician ought to be aware of their indications of use and potential side effects.

Question 7 of 15 Which of the following is correct regarding transmission, prevention and treatment of avian 'bird' flu (H5N1)? (Please select 1 option) Bird flu is not transmitted by touching birds dead or alive Bird flu is not transmitted from being in contact with dried bird droppings of infected birds Bird flu is not transmitted through cooked food It is suggested that travel to countries affected by bird flu be avoided Oseltamivir (Tamiflu) and zanamivir (Relenza) will cure a person infected with bird flu

Bird flu is not transmitted through cooked food Correct There are many types of bird flu, most of which are harmless to humans. However a couple of strains have caused concern globally in recent years, affecting large numbers of humans and threatening pandemic levels. The World Health Organization (WHO) has confirmed that as of July 2013, 633 people had been infected with the H5N1 virus and 377 had died. Between 2003 and July 2013 WHO has confirmed cases of H5N1 in humans in: Azerbaijan Bangladesh Cambodia China (outbreaks of H7N9 strain also reported) Djibouti Egypt Indonesia Iraq Laos Myanmar (Burma) Nigeria Pakistan Thailand Turkey, and Vietnam. Tamiflu and Relenza (in pregnancy) are licensed as antivirals for avian flu. However these may (or may not) help to alleviate symptoms; they are by no means prescribed as a 'cure'. Treatment is largely supportive with fluids and paracetamol. In cases of super-added pneumonia targeted treatment of staphylococcal infection is required. Reference: NHS Choices. Bird flu (avian flu).

Question 22 of 31 A 40-year-old female presents with reduced vision. Her husband mentions that she recently started 'bumping into things'. Fundoscopy did show temporal pallor in both optic discs. Her MRI scan is shown below: What field defect would you expect her to have from her MRI scan? (Please select 1 option) Bitemporal hemianopia Bilateral enlarged blind spot Left junctional scotoma Left homonymous hemianopia Right superior quadrantanopia

Bitemporal hemianopia This is the correct answer The MRI image shows a large pituitary lesion compressing the optic chiasm. Therefore, the resultant field defect would be a bitemporal hemianopia since the nasal fibres are compressed. Because the chiasm is part of the optic tract temporal pallor, reduced colour vision and a relative afferent pupillary defect may also be present. A pituitary lesion needs to be greater than 10 mm to give rise to compression of the chiasm. A bilateral enlarged blind spot may be present in cases of bilateral disc swelling caused by papilloedema. Left junctional scotoma is present in cases of a lesion adjacent to the optic chiasm and the opposite optic nerve. It is usually caused by a meningioma. Left homonymous hemianopia is caused by a lesion in the contralateral parietal region of the brain, i.e. right parietal area. Right superior quadrantanopia is caused by the contralateral temporal region in the area known as Meyers loop, i.e left temporal region. Note the age of the patient when there are questions concerning field defects, since this will give you a clue with regards to aetiology: Young age - craniophrangioma Middle age - pituitary or meningioma Elderly - likely to be stroke related.

Question 21 of 31 A 36-year-old female presents with blurry vision. She has had no systemic symptoms upon questioning. Upon ophthalmoscopy you notice the following changes below: What would be your first investigation? (Please select 1 option) Blood pressure Computerised Tomography Lumbar Puncture T2 weighted MRI Urine dipstick

Blood pressure This is the correct answer Bilateral disc swelling with nerve fibre flame haemorrhages is strongly associated with malignant hypertension. She also has no symptoms reported to suggest idiopathic intracranial hypertension. Always consider malignant hypertension as a cause of bilateral disc swelling. Checking the patient's blood pressure will alleviate the need for unnecessary neuroimaging in these patients. If BP is normal then it would be feasible to order a computerised tomography scan or MRI as the next investigation. However, the question asked the first investigation. Lumbar puncture should never be undertaken unless neuroimaging has been undertaken to rule out a space occupying lesion. Urine dipstick may be abnormal with high BP .

Question 10 of 22 You are on a post-take ward round. A 32-year-old Asian man has been admitted for alcohol withdrawal and a possible left leg deep vein thrombosis (DVT). On examination he has extensive tattoos on his arms, you also notice some scratch marks on the arms. There is epigastric tenderness in the abdomen. He is very thin and admits to significant weight loss over the past two months, he is unable to quantify the exact amount and attributes this to symptoms of anorexia and vomiting, secondary to alcohol use. The patient reports that his stools and urine have been normal, though you note dark urine in a bottle next to the bed. Blood tests are as follows: Hb 120 g/L WCC 6.3 ×109/L Platelets 480 ×109/L Lactic acid 6.0 mmol/L Albumin 28 g/L ALT 623 IU/L Alk Phos 186 IU/L Bilirubin 76 µmol/L CRP 56 mg/L He has been commenced on an alcohol detox regime and treatment dose low molecular weight heparin. Which of the following would you wish to add to his management plan? (Please select 1 option) CT abdomen CEA Hepatitis screen HIV Liver ultrasound

CT abdomen This is the correct answer Weight loss, jaundice and abdominal pain are red flags for pancreatic cancer. Associated and unexplained thromboses may occur, recognised to be "Trousseau's sign". Pancreatic cancer is notoriously difficult to diagnose. The clinical presentation is not consistent with a simple alcohol withdrawal and there is no obvious reason for this man to sustain a DVT. Some alarm signs include a cholestatic picture of liver derangement, rather than a purely hepatic picture which would be expected in hepatitis. A very low albumin may indicate the presence of an underlying catabolic process in the body, whilst a high lactate may suggest cell necrosis. A CT of the abdomen would be a sensible form of imaging to start with, ultrasound is not good for retroperitoneal structures such as the pancreas. In cases where the tumour is sited in the head of the pancreas there may be associated hyperbilirubinaemia, pruritis, pale stools and dark urine due to blockage of the bile duct. Some of these signs are present here. Have a low threshold to investigate, even though the patient is young.

Question 11 of 22 Core Questions A 49-year-old male patient with chronic liver disease and known liver cirrhosis secondary to alcohol abuse, is admitted through the Emergency department. He is confused and has suffered with an altered sleep pattern for the past forty eight hours. On examination the Glasgow coma scale is 13 (E2, V5, M6). He is apyrexial. His abdominal examination reveals a soft abdomen with no obvious ascites. There are stigmata of chronic liver disease. Blood tests are as follows: Hb 135 g/L WCC 7 ×109/L Platelets 128 ×109/L Na 131 mmol/L K 3.6 mmol/L Creatinine 49 μmol/L Urea 4 mmol/L Magnesium 0.81 mmol/L Serum ammonia is within normal limits. How would you proceed to manage this patient? (Please select 1 option) CT brain IV fluids Lactulose Lumbar puncture Rifaximin

CT brain This is the correct answer The question here leads the reader to believe there may be a hepatic encephalopathy present. However, the serum ammonia is within normal limits. The abdomen is also soft and non-tender, and inflammatory markers are low, thus excluding spontaneous bacterial peritonitis as a source of altered GCS. So, despite the history, there is now a man with an altered conscious level and no obvious cause. A CT head is entirely appropriate and must be performed first line in order to exclude an acute intracranial abnormality. You should have a low threshold to image the brain in a confused patient, even with a background of alcohol or toxin abuse; it is safer to exclude intracranial abnormality before treating for a presumed diagnosis.

CVS

CVS

Question 6 of 22 A 57-year-old man with a history of cirrhosis presents with high grade fever, abdominal pain and a peripheral leucocytosis. Ultrasound of the abdomen confirms cirrhosis and diagnoses the presence of ascites. There are no abdominal collections of note, the kidneys look normal. Chest x ray shows clear lung fields. ECG reveals a sinus tachycardia of 120/min. Urine dip is negative. There is no history of drug allergy and renal function is normal. Which antibiotic would be appropriate to start empirically in this man? (Please select 1 option) Cefotaxime 2 g intravenously Ciprofloxacin 500 mg intravenously Levofloxacin 500 mg intravenously Metronidazole 500 mg intravenously Amoxicillin 1g intravenously

Cefotaxime 2 g intravenously This is the correct answer Most cases of spontaneous bacterial peritonitis (SBP) are due to gut bacteria such as Escherichia coli and Klebsiella. Less commonly streptococcal and staphylococcal infections may also occur. Treatment should be instituted early, ideally with a third generation cephalosporin such as cefotaxime, following ascitic tap. A diagnosis of spontaneous bacterial peritonitis should be made with a low threshold for anybody with ascites. Presenting symptoms often include pyrexia and abdominal pain and a white cell count of greater than 250 cells/mm3 on ascitic tap. Sending the fluid sample for culture will help to identify the bacteria and its sensitivities. Many patients with cirrhosis are maintained on long term antibiotics, usually a fluoroquinolone, as prophylaxis for SBP. It is best to avoid using a fluoroquinolone for treatment in these cases as resistance may have developed to the drug. Levofloxacin may be used in patients with penicillin allergy, however clinical evidence for its efficacy is low. Empiric antibiotic treatment may be discontinued after 48 hours if cultures show no bacterial growth. Reference: McHutchison JG, Runyon BA. Spontaneous bacterial peritonitis. In: Surawicz CM, Owen RL (eds.) Gastrointestinal and Hepatic Infections. Philadelphia; Saunders: 1995. p 455.

Question 28 of 31 Core Questions An 83-year-old woman presents with complete loss of vision to the right eye. She has been complaining of muscle aches in her thighs recently which her GP attributed to her age. She also mentions pain upon talking and weight loss. Her ESR is 10 and CRP 32. What is your immediate management? (Please select 1 option) Arrange temporal artery biopsy Commence methylprednisolone intravenously Commence oral prednisolone (0.5 mg/kg) Commence oral prednisolone (1 mg/kg) Repeat ESR and CRP Test Analysis

Commence methylprednisolone intravenously This is the correct answer Symptoms and signs are consistent with that of giant cell arteritis (GCA). Loss of vision is due to arteritic ischaemic optic neuropathy and lingual artery involvement. There is also polymyalgia rheumatica (PMR) as evidenced by the lower pelvic stiffness and weight loss. There is a 40% association with GCA and PMR. Both can also present with a normal ESR or CRP, so-called occult GCA which can occur in up to 20% of cases. Since there is ophthalmic involvement the risk of second eye involvement is high, up to 50-90%. Intravenous initiation of methylprednisolone is preferred as opposed to oral steroid. Treatment should be initiated before biopsy is arranged since this is an ophthalmic emergency.

Question 31 of 31 A 46-year-old man with a history of multiple sclerosis (MS) is re-admitted to hospital out of hours as a failed discharge. He has been treated with pulsed methylprednisolone for three days due to a flare up of his symptoms. He had seen some mild improvement and had been discharged home with physiotherapy and social services support that day. However he has been unable to cope at home and the glucocorticoid therapy has failed to improve muscle strength as anticipated. What would you advise? (Please select 1 option) Add oral prednisolone in tapering dose Continue high dose methylprednisolone 1 g IV to day 5 Glatiramer acetate Interferon gamma Plasma exchange therapy

Continue high dose methylprednisolone 1 g IV to day 5 This is the correct answer Recommended first line therapy for relapsing MS is five days of methylprednisolone 1 g intravenously. For patients with an acute multiple sclerosis exacerbation that results in neurologic symptoms and increased disability or impairments in vision, strength, or cerebellar function, recommended treatment is with glucocorticoids. The preferred regimen is intravenous methylprednisolone 1 g daily for five days without an oral taper. Repository corticotropin injection gel, where available, is an alternative for patients with MS exacerbations who cannot tolerate high-dose glucocorticoids or have poor venous access or prefer self-injection. Infection must be ruled out first. For patients with acute, severe neurologic deficits caused by multiple sclerosis who have a poor response to treatment with high-dose glucocorticoids, suggested treatment is with plasma exchange. Patients with relapsing-remitting MS who have current disease activity manifested by clinical symptoms or MRI lesions should be offered treatment with disease-modifying therapy, such as an interferon or glatiramer acetate. Reference: Thompson AJ, Kennard C, Swash M, et al. Relative efficacy of intravenous methylprednisolone and ACTH in the treatment of acute relapse in MS. Neurology. 1989;39:969-71.

Question 17 of 22 Core Questions A 24-year-old building contractor is admitted with coffee ground vomiting. He has a history of 14 units of alcohol consumption per week and has recently started taking ibuprofen for back pain. On examination his blood pressure is 138/80 mmHg and heart rate is 86 bpm. There is some epigastric tenderness in the abdomen but no melaena on digital examination. His blood results show: Hb 140 g/L WCC 6 ×109/L Plts 274 ×109/L His renal function including urea is normal. He has no past medical history. Which of the following is the most appropriate management? (Please select 1 option) Arrange an inpatient liver USS, LFTs, and follow up in outpatient clinic Commence a proton pump inhibitor and observe as an inpatient for 24 hours Discharge home with a proton pump inhibitor and an outpatient endoscopy Give IV fluids, an IV PPI, and arrange an inpatient OGD Keep NBM for inpatient OGD and discharge if normal

Discharge home with a proton pump inhibitor and an outpatient endoscopy This is the correct answer The Glasgow-Blatchford score for this patient is zero, which puts him in the low risk group for a significant bleed. Therefore it would be appropriate to manage him as an outpatient. In the meantime, given the history of NSAID usage, it may be wise to substitute his choice of analgesic and commence an oral proton pump inhibitor. Criteria for Glasgow-Blatchford score: Marker Risk Score Blood urea (mmol/L) 6.5-7.9 2 8.0-9.9 3 10.0-25.0 4 >25 6 Haemoglobin (g/L) for men 120-129 1 100-119 3 <100 6 Haemoglobin (g/L) for women 100-109 1 <100 6 Systolic blood pressure (mm Hg) 100-109 1 90-99 2 <90 3 Other markers Pulse ≥100 (per min) 1 Presentation with melaena 1 Presentation with syncope 2 Hepatic disease 2 Cardiac failure 2 Low-risk criteria of GBS: urea <6.5 mmol/L haemoglobin ≥130 g/L (men) or ≥120 g/L (women) systolic blood pressure ≥110 mmHg pulse <100 beats per min, and absence of melaena, syncope, cardiac failure, or liver disease. In the validation group, scores of 6 or more were associated with a greater than 50% risk of needing an intervention. This score has replaced the previously used Rockall score, as it can be calculated without OGD findings and is deemed to be more objective. Reference: Stanley AJ, Ashley D, Dalton HR, et al. Outpatient management of patients with low-risk upper-gastrointestinal haemorrhage: multicentre validation and prospective evaluation. Lancet. 2009;373:42-7.

A 56-year-old man presents with acute onset of shortness of breath and chest pain. On examination he looks unwell. He is tachypnoiec 32/min, temperature is 37.3°C, heart rate 120/min, and blood pressure 89/48 mmHg. He has recently disembarked from a long haul flight and you strongly suspect a pulmonary embolus. Due to the haemodynamic instability you request an urgent bedside ECHO. Which of the following would confirm presence of acute right ventricular strain on the ECHO? (Please select 1 option) Left ventricular wall motion abnormality Pericardial effusion Reduced right ventricular systolic function Right atrial diastolic collapse Right ventricular diastolic collapse

ECHO findings in massive pulmonary embolism would demonstrate signs of acute ventricular strain, including right ventricular dilatation and reduced right ventricular systolic function. In a massive PE causing haemodynamic instability bedside ECHO is the safer option compared to transferring to radiology for CT pulmonary angiogram. Evidence of acute right ventricular overload with a history and examination consistent for pulmonary embolism should be sufficient to commence treatment. ECHO findings in a massive PE include: Right ventricular dilatation, often larger than the left ventricle Reduced right ventricular systolic function, and McConnell's (hinge) sign, which demonstrates relative sparing of the right ventricular apex. Treatment modalities for PE range through heparinisation, warfarinisation, thrombolysis and embolectomy. The treatment algorithm below provides a useful summary: Reference: Torbicki A, Perrier A, Konstantinides S, et al. Guidelines on the diagnosis and management of acute pulmonary embolism. Eur Heart J. 2008;29(18):2276-315.

Question 4 of 15 A 19-year-old university student has been brought to the outpatient clinic by her mother because of persistent symmetrical enlarged lumps in the neck. An examination reveals a slim girl with enlarged parotid glands. She is bradycardic and hypotensive with a postural drop and has cool peripheries with a slow capillary refill. You take some blood which shows hypokalaemia, high cholesterol and high cortisol levels. The mother thinks that the lumps were not there when she was living at home and that they are caused by stress from university. What is the most likely diagnosis? (Please select 1 option) Conn's syndrome Cushing's syndrome Eating disorder Hodgkin's lymphoma Toluene intoxication

Eating disorder This is the correct answer Features of eating disorders include low BMI, bradycardia, hypotension with postural hypotension if severe. Swollen salivatory glands are especially, but not exclusively, seen in patients with bulimia. The findings in the blood include hypokalaemia, low follicle-stimulating hormone (FSH), luteinising hormone (LH), sex hormones, low tri-iodothyronine (T3), and high cortisol, cholesterol, growth hormone and glucose (impaired glucose tolerance). You can remember this as Gs & Cs raised (that is, glucose, growth hormone, cortisol, cholesterol), but everything else low. Cushing's and Conn's can have features described in the blood results but the patient would characteristically be hypertensive. Toluene intoxication is otherwise known as 'glue sniffing' where a neurological pathology would be the predominant feature. Reference: Nicholls D, Hudson L, Mahomed F. Managing anorexia nervosa. Arch Dis Child. 2011;96:977-82. Walsh BT, Croft CB, Katz JL. Anorexia nervosa and salivary gland enlargement. Int J Psychiatry Med. 1981;11:255-61.

A 25-year-old man is admitted to the Emergency department after developing sharp left sided chest pain at work and he is breathless. He works as a removals man. He is a non-smoker and has no past history of note. His x ray is shown below: What would be the most appropriate next step, in line with the current BTS guidelines? (Please select 1 option) Emergency needle decompression of left hemithorax Intercostal Seldinger chest drain insertion with underwater seal Large diameter surgically inserted chest drain Needle aspiration of at least 2.5 litres and reassessment Observe and repeat x ray in one week

Emergency needle decompression is reserved for a tension pneumothorax, or large pneumothoraces showing signs of 'pressure' such as mediastinal shift. In this case the mediastinum has shifted over the to the right due to the large pnemothorax in the left hemithorax. Typically to decompress the pneumothorax, a wide-bore needle is placed in the second rib space and a characteristic hissing noise should be heard, indicating release of pressure. Immediately after a chest drain ought to be sited, to prevent re-acculumation of air. For large pneumothoraces with absence of pressure signs, the current BTS guidelines for management advise the following: In a primary/spontaneous pneumothorax for a patient with no other lung diagnoses aged under 50, needle aspiration of at least 2.5 litres is the first line treatment. Observation alone is unlikely to reinflate the lung with such a large collapse. A chest drain may be needed if aspiration is not successful but should not be first line due to the increased risk of organ damage and pain. A large diameter drain is generally not required except in some cases of pus/loculated effusions and after surgical procedures. Reference: MacDuff A, Arnold A, Harvey J. Management of spontaneous pneumothorax: British Thoracic Society Pleural Disease Guideline 2010. Thorax. 2010;65:ii18-31.

Question 7 of 31 A patient is complaining of headache post-lumbar puncture (LP) which was conducted approximately 48 hours before. Simple measures have been tried but the headache is not resolving despite bed rest, regular analgesia and oral fluids. You suspect it is secondary to low pressure. Which of the following is the treatment of choice? (Please select 1 option) Epidural blood patch Epidural fibrin glue Intravenous caffeine Intravenous rehydration Opioid analgesia

Epidural blood patch This is the correct answer Post-lumbar puncture headache is relatively common. Epidural blood patch remains the treatment of choice if the headache fails to resolve with conservative management. Use of an atraumatic spinal needle (22 gauge Quincke), is advised to try and prevent this complication. Post-lumbar puncture headache occurs in 10-30% of patients undergoing a LP. In a large proportion the headache will resolve spontaneously or with conservative measures. However a significant proportion may continue to have a headache for days afterwards. The treatment of choice is an epidural blood patch. UpToDate state the following: "A systematic review published in 2010 concluded that epidural blood patch is effective compared with both conservative treatment and sham procedure, based upon the results of three randomized controlled trials with a total of 86 subjects. The first epidural blood patch gives relief in the majority of cases, and the second one is successful in nearly all cases. Thus, epidural blood patch is considered the treatment of choice for patients with PLPHA who fail an initial trial of conservative management." Surgical repair of the CSF leak may be required in some patients if all else fails. Reference: Schievink WI. Spontaneous spinal cerebrospinal fluid leaks and intracranial hypotension. JAMA. 2006;295:2286-96. Seebacher J, Ribeiro V, LeGuillou JL, et al. Epidural blood patch in the treatment of post dural puncture headache: a double blind study. Headache. 1989;29:630-2. Strupp M, Schueler O, Straube A, et al. "Atraumatic" Sprotte needle reduces the incidence of post-lumbar puncture headaches. Neurology. 2001;57:2310-2. UpToDate. Post-lumbar puncture headache. van Kooten F, Oedit R, Bakker SL, Dippel DW. Epidural blood patch in post dural puncture headache: a randomised, observer-blind, controlled clinical trial. J Neurol Neurosurg Psychiatry. 2008;79:553-8.

A 42-year-old woman presents to the Emergency department with an episode of collapse. She was apparently sitting in local coffee shop when she felt the onset of very rapid palpitations and then fell to the floor. She has a history of asthma for which she takes a regular Seretide inhaler and has just been prescribed a course of erythromycin for a lower respiratory tract infection. It appears that she also purchased an over-the-counter fluconazole tablet for an episode of vulval candida infection that morning. Examination reveals a BP of 122/72 mmHg, pulse is 75 and regular. There are no murmurs and her chest appears clear. As you are about to walk away from the bedside you notice a paroxysm of 10-12 sec of torsades de pointes VT on the monitor. Investigations reveal: Hb 128 g/L (135-180) WCC 6.2 ×109/L (4.5-10) PLT 230 ×109/L (150-450) Na 136 mmol/L (135-145) K 4.1 mmol/L (3.5-5.5) Cr 82 µmol/L (70-110) Magnesium 0.9 mmol/L (>0.7) ECG shows sinus rhythm with a QT interval of 610 ms. Which of the following is the most appropriate intervention? (Please select 1 option) Amiodarone Bisoprolol Isoprenaline Magnesium Overdrive pacing

Even though the magnesium level is above the lower limit of normal, studies still indicate that magnesium infusion increases the chances of maintaining sinus rhythm in patients presenting with torsades. It can be given as a bolus of 1-2 g/IV initially over 30-60 seconds, or as an infusion at a rate of 3-10 mg/min. Close monitoring is required because of the risk of inducing depression of neuromuscular function via hypermagnesaemia. Amiodarone and bisoprolol can potentiate the risk of torsades via further QT prolongation and must therefore be avoided in this situation. Isoprenaline can be used to increase the ventricular rate in sinus rhythm to above 90 bpm, reducing the risk of further episodes of torsades. In the event that isoprenaline is ineffective, overdrive pacing to increase the ventricular rate can be considered. In this situation it is likely that washout of the fluconazole and macrolide will lead to cessation of the episodes of torsades.

A 36-year-old man presents to the Emergency department with palpitations and pre-syncope. He is a fit and healthy long distance runner who has completed a half marathon some 24 hours earlier. He admits to having celebrated the previous evening, having drunk up to six pints of beer. He has no past medical history of note and completed an insurance medical including echocardiography only one year earlier. Examination reveals a BP of 120/70 mmHg, pulse is 165 (atrial fibrillation). His chest is clear. Investigations reveal: Hb 142 g/L (135-180) WCC 6.8 ×109/L (4.5-10) PLT 203 ×109/L (150-450) Na 137 mmol/L (135-145) K 4.0 mmol/L (3.5-5.5) Cr 102 µmol/L (70-110) Glucose 4.2 mmol/L - Troponin I <0.05 (<0.05) ECG shows fast atrial fibrillation ventricular rate 160. Which of the following is the most appropriate intervention for managing the AF? (Please select 1 option) Amiodarone Bisoprolol DC cardioversion Digoxin Flecainide

Flecainide delivered IV is highly effective in converting paroxysmal atrial fibrillation. However, the results of the CAST trial, which showed increased mortality in association with its use post-myocardial infarction, have restricted its use to patients without a history of ischaemic or structural heart disease. It is given as a loading dose of up to 150 mg over the first 30 minutes, and then at a slower rate for the next hour, and a maintenance rate for the remainder of the first 24 hours, adding up to a cumulative dose of 600 mg. Transition to oral medication is recommended as soon as possible. It is important to remember to anticoagulate the patient using low molecular weight heparin when chemically cardioverting using this drug. In the presence of structural or ischaemic heart disease, amiodarone IV would be the first choice option for chemical cardioversion. In head-to-head trials it is marginally less effective than flecainide with respect to attaining sinus rhythm. Bisoprolol is recommended as oral therapy by NICE guidelines for maintenance of sinus rhythm in patients with a history of paroxysmal AF. In this situation digoxin is a suboptimal choice because it will merely reduce ventricular rate rather than attaining sinus rhythm. In this situation with no signs of cardiac failure and preserved BP, there is no indication to progress to DC cardioversion as an emergency procedure. Reference: Electronic Medicines Compendium (eMC). Tambocor 10 mg/ml Injection. NICE. Atrial fibrillation: the management of atrial fibrillation (CG180).

A 51-year-old man presents to you with acute shortness of breath. This condition has been deteriorating over several years, so that he is now limited to walking on the flat. His past medical history is unremarkable apart from a history of a 'severe chest infection' aged 21 for which he required intravenous antibiotics as an inpatient. His chest x ray is reported as normal. Basic spirometry tests are as follows: FEV1 2.1 litres (52% predicted) FVC 4.9 litres (103% predicted) FEV1/FVC ratio 0.5 He is a long term smoker. Which of the following should be recommended? (Please select 1 option) Home pulmonary rehabilitation Home salbutamol nebulisers Initiation of inhaled long acting steroid Initiation of muscarinic antagonist Long term macrolide

For a patient with obstructive spriometry or presumed COPD and an FEV1 over 50% predicted, a first line treatment would be inhaled short acting beta agonist plus either inhaled long acting beta agonist or long acting muscarinic antagonist. The spirometry shows an obstructive pattern. In the context of long term smoking the most likely explanation is COPD although it may be reasonable to rule out reversibility to inhaled bronchodilators first to exclude asthma. The GOLD criteria for COPD score severity into the following categories based on FEV1: Mild - 80-100% Moderate - 50-80% Severe - 30-50% Very severe - under 30%. This man would therefore be treated as moderate. He is not severe enough to be given home nebulisers, and there is very little evidence for home pulmonary rehabilitation being effective at all. There is no evidence for inhaled steroids alone in COPD, unlike asthma. We do not know his exacerbation frequency to suggest that macrolides would benefit him. Reference: NICE. Chronic obstructive pulmonary disease in over 16s: diagnosis and management (CG101).

Question 13 of 22 A 24-year-old building contractor is admitted with coffee ground vomiting. He has a history of 14 units of alcohol consumption per week and has recently started taking ibuprofen for back pain. On examination his blood pressure is 99/60 mmHg and his heart rate is 110 bpm. There is some epigastric tenderness in the abdomen and malaena on digital examination. Blood results show: Hb 120 g/L WCC 6 ×109/L Plts 274 ×109/L Renal function reveals a raised urea of 16 mmol/L. He has no past medical history. What is the most appropriate management? (Please select 1 option) Arrange an inpatient liver USS, LFTs and follow up in outpatient clinic Commence a proton pump inhibitor and observe as an inpatient for 24 hours Discharge home with a proton pump inhibitor and an outpatient endoscopy Give IV fluids to resuscitate the patient and arrange an inpatient OGD Keep NBM for inpatient OGD and discharge if normal

Give IV fluids to resuscitate the patient and arrange an inpatient OGD This is the correct answer This man has a Glasgow-Blatchford score (GBS) of 6, which indicates 50% risk of needing an intervention. Criteria for Glasgow-Blatchford: Marker Risk Score Blood urea (mmHg) 6.5-7.9 2 8.0-9.9 3 10.0-25.0 4 >25 6 Haemoglobin (g/L) for men 120-129 1 100-119 3 <100 6 Haemoglobin (g/L) for women 100-109 1 <100 6 Systolic blood pressure (mmHg) 100-109 1 90-99 2 <90 3 Other markers Pulse ≥100 (per min) 1 Presentation with melaena 1 Presentation with syncope 2 Hepatic disease 2 Cardiac failure 2 Low risk criteria of GBS: urea <6.5 mmol/L haemoglobin ≥130 g/L (men) or ≥120 g/L (women) systolic blood pressure ≥110 mmHg pulse <100 beats per min absence of melaena, syncope, cardiac failure, or liver disease. In the validation group, scores of 6 or more were associated with a greater than 50% risk of needing an intervention. This score has replaced the previously used Rockall score, as it can be calculated without OGD findings and is deemed to be more objective. Reference: Stanley AJ, Ashley D, Dalton HR, et al. Outpatient management of patients with low-risk upper-gastrointestinal haemorrhage: multicentre validation and prospective evaluation. Lancet. 2009;373:42-7.

A 62-year-old man is admitted to the Emergency department with an episode of ventricular tachycardia. He collapsed in the local supermarket. The VT was captured on the ambulance recording strip, and he reverted spontaneously. He was discharged from the coronary care unit after suffering an extensive anterior MI (for which he received two stents), some two weeks earlier. He has been prescribed multiple medications including ramipril 10 mg, furosemide 40 mg and bisoprolol 5 mg. Examination reveals a BP of 105/80 mmHg, pulse is 84 and regular. There are bilateral basal crackles consistent with heart failure, and bilateral pitting oedema of both ankles. Investigations reveal: Hb 134 g/L (135-180) WCC 7.0 ×109/L (4.5-10) PLT 200 ×109/L (150-450) Na 140 mmol/L (135-145) K 5.2 mmol/L (3.5-5.5) Cr 132 µmol/L (70-110) ECG shows deep anterior Q waves, 1-2 mm of ST elevation in V2-V5. CXR shows LVH. ECHO shows evidence of possible LV aneurysm, ejection fraction 22%. Which of the following is most appropriate intervention with respect to the VT? (Please select 1 option) Implantable cardioverter defibrillator Increased dose of bisoprolol Oral amiodarone Outpatient Holter monitor Surgical excision of aneurysm

Given this patient's ejection fraction, and possible LV aneurysm, his prognosis with respect to further episodes of VT and sudden cardiac death is very poor. As such progression to implantable cardioverter defibrillator (ICD) insertion at an early stage is recommended. Increased dose of bisoprolol will not significantly reduce the likelihood of further episodes of VT in this situation in light of the underlying LV aneurysm and very poor ejection fraction. In studies oral amiodarone has proven inferior to ICD implantation with respect to survival and as such is not the preferred option here. Arranging an outpatient Holter monitor risks a potentially fatal VT episode whilst the investigation is being arranged. Aneurysm excision is reserved for those with recurrent VT despite other interventions. The patient should of course be assessed for the presence of intraventricular thrombus, as anticoagulation may be required.

A 52-year-old man is admitted in the early hours of the morning after suffering palpitations and presyncope. He is usually well and still plays veterans rugby. He had been at a party the previous evening during which he consumed seven pints of beer. He smokes 5 cigars per week. Examination reveals a BP of 105/72 mmHg, pulse of 130 and atrial fibrillation. His chest is clear. Investigations reveal: Hb 138 g/L (135-180) WCC 7.8 ×109/L (4-10) PLT 302 ×109/L (130-400) Na 136 mmol/L (135-145) K 3.7 mmol/L (3.5-5.5) Cr 112 µmol/L (70-120) Troponin <0.05 - ECG shows atrial fibrillation, no other acute changes. CXR shows normal heart size, no evidence of heart failure. Which of the following is the most appropriate intervention? (Please select 1 option) DC cardioversion IV adenosine IV amiodarone IV flecainide Oral sotalol

Given this person has had no oral intake over the past few hours, and is young without a previous cardiac history, DC cardioversion is the most appropriate next step, as the chances of achieving permanent sinus rhythm are high. Binge drinking is well known as a potential trigger for paroxysmal AF. He should of course be given advice to moderate his alcohol consumption in future. IV flecainide is associated with higher rates of conversion to sinus rhythm than amiodarone, and is the preferred choice for chemical cardioversion in patients with a structurally normal heart who have no previous history of IHD. Where there is some debate about duration of AF (that is, potentially >24 hours), anticoagulation is instituted prior to cardioversion because of the risk of precipitating emboli after sinus rhythm is attained. In this situation we can't exclude structural or ischaemic heart disease, and therefore DC cardioversion is preferred to flecainide. Over the longer term, bisoprolol is indicated by NICE guidance as the first line prophylaxis against further episodes of paroxysmal AF. Adenosine has a role in terminating AV nodal re-entrant tachycardias and may be useful as a diagnostic aid in other paroxysmal SVTs. Reference: NICE. Atrial fibrillation: the management of atrial fibrillation (CG180).

Question 12 of 31 A 36-old-male recently diagnosed with HIV has a CD4 count of 100. He is asymptomatic with regard to his vision. Upon ophthalmoscopy you notice the following changes in both his eyes: What is the likely cause for fundal changes? (Please select 1 option) CMV retinitis HIV related retinopathy Progressive outer retinal necrosis (PORN) Purtscher retinopathy Toxoplasmosis chorioretinitis

HIV related retinopathy This is the correct answer The fundal image displays multiple cotton wool spots which is consistent with that of the micro-ischaemic changes found in patients with HIV. These patients are generally asymptomatic. It occurs in 50-60% of patients with advanced HIV. CMV retinitis is incorrect here as the fundal picture does not show the characteristic pizza pie and tomato ketchup appearance, which represents the exudation and haemorrhage. Additionally, patients may also be symptomatic. CMV retinitis should be considered if the CD4 count is less than 50. Purtscher retinopathy is associated with patients who have sustained a head injury or blunt thoracic trauma. The retinopathy is similar with regard to cotton wool spots. Progressive outer retinal necrosis (PORN) is found amongst HIV patients with very low CD counts. The lesions are found within the macular, hence vision is significantly affected with these patients. With Toxoplasma chorioretinitis there may be a pigmented toxoplasmosis lesion or multiple raised choroidal lesions (primary toxoplasmosis). Patients also tend to have a dense vitritis and are symptomatic of floaters.

Question 2 of 15 A 63-year-old widow who is recovering from a hip fracture has been brought to the Emergency department by her daughter. Her daughter says she has been behaving bizarrely. She cannot hold a conversation without forgetting the topic, and has been unnecessarily angry. She says she has woken up on the couch a lot over the last few days without noticing that she had fallen asleep. On examination she is disorientated to time and place. The daughter visits her every month as her mother lives alone. On her previous visit her mother was 'absolutely fine'. What factor puts this patient at increased risk of suffering from delirium? (Please select 1 option) Age Hip fracture Living alone Marital status Sex

Hip fracture This is the correct answer Patients with the following are at increased risk of delirium: 65 years or older cognitive impairment current hip fracture, and severe illness. Acute decline in cognitive ability, hallucinations, reduced physical function, altered social behaviour, change in mood, and change in concentration levels, especially being more sedated all point towards a diagnosis of delirium. Delirium can be differentiated from dementia due to its rapid onset and deterioration. In this case the symptoms were not seen one month previously. Reference: NICE. Delirium: prevention, diagnosis and management (CG103).

Question 11 of 15 A junior doctor in the Emergency department asks for your help with the rapid tranquilisation of a patient suffering from pneumonia. The patient has become aggressive, randomly shouting bizarre noises and trying to pull out his cannula. A nurse tried to calm him down by reassuring and orientating him but he grabbed on to her uniform and tried to rip it off. It has been documented in the past that he can become delirious and that it can be difficult to administer oral medication. What would be your choice of drug in this case? (Please select 1 option) IM lorazepam IM lorazepam and IM haloperidol IV midazolam Oral lorazepam Oral lorazepam and oral haloperidol

IM lorazepam and IM haloperidol This is the correct answer Rapid tranquilisation, physical intervention and seclusion should only be considered once de-escalation and other strategies have failed to calm the patient. Oral therapy should always be first line; however, if this has been refused or a proportionate response has not been reached, IM should be the next step. In this case, de-escalation techniques have resulted in potential harm to a staff member and it is very unlikely that the patient will accept any medication orally. If a patient has a psychotic context to his symptoms it would be appropriate to give an anti-psychotic as an add-on to a benzodiazepine. The Royal College of Psychiatrists recommend lorazepam and haloperidol. Procyclidine or benzotropine should be immediately available to reduce the risk of dystonia or other extrapyramidal side effects. IV would be the last resort, and should be used if immediate tranquilisation is essential. Reference: NICE. Violence and aggression: short-term management in mental health, health and community settings (NG10). Royal College of Psychiatrists. Rapid tranquillisation (RT) algorithm.

Question 3 of 22 Core Questions A 42-year-old alcoholic is admitted by emergency ambulance after vomiting approximately 250 ml of fresh blood. He is known to have oesophageal varices and failed to attend an appointment for prophylactic banding some three months earlier. He is poorly compliant with other therapies including propranolol and spironolactone. On examination his BP is 105/70 mmHg, pulse is 95 and regular. There is tenderness in the epigastrium. Investigations show: Hb 84 g/L (130-180) WCC 10.5 ×109/L (4-10) PLT 95 ×109/L (130-400) Na 137 mmol/L (135-145) K 4.9 mmol/L (3.5-5.5) Cr 128 µmol/L (70-120) Urea 14.1 mmol/L (2.5-8) INR 1.2 - There is an infusion of normal saline running. Which of the following is the most important next step? (Please select 1 option) Injection sclerotherapy IV antibiotics IV omeprazole IV terlipressin TIPS

IV terlipressin This is the correct answer In this situation IV terlipressin is proven to impact positively on mortality in acute variceal bleeding. It reduces mortality by 34% versus placebo. It has also been shown to reduce failure of haemostasis by 25% in patients who undergo variceal banding. Although mortality is low in banded patients and therefore numbers in studies are small, significance for benefit of terlipressin vs. sclerotherapy alone was nearly achieved. For these reasons NICE recommend it is started as soon as possible after presentation with variceal haemorrhage. Injection sclerotherapy is recommended for gastric varices. In oesophageal variceal bleeding, banding is associated with a better outcome. Prophylactic antibiotics should be started, but in this situation IV terlipressin is more pressing. There is no evidence for using IV PPI for variceal haemorrhage, and TIPS is reserved for patients where haemostasis is not achieved despite banding. Reference: Ioannou GN, Doust J, Rockey DC. Systematic review: terlipressin in acute oesophageal variceal haemorrhage. Aliment Pharmacol Ther. 2003;17:53-64. NICE. Acute upper gastrointestinal bleeding in over 16s: management (CG141).

A 70-year-old lady re-presents to hospital two weeks after a short, three day admission during which she was treated for a CURB score 1 right basal community acquired pneumonia, with a seven day course of amoxicillin and clarithromycin. At that time, blood/sputum cultures and urinary antigen tests for Legionella and Pneumococcus were negative. She now has a persistent cough and new, right-sided chest discomfort. Her past history includes type 2 diabetes mellitus, hypertension, rheumatoid arthritis and a chronic right-sided pleural effusion. She is comfortable at rest, with oxygen saturations 94% on room air, respiratory rate 16/min, blood pressure 135/77 mmHg, heart rate of 78 bpm and temperature of 37.3°C. Investigations reveal: Haemoglobin 134 g/L (115-165) White cell count 10.8 ×109/L (4.0-11.0) Platelet count 185 ×109/L (150-400) Serum sodium 136 mmol/L (137-144) Serum potassium 4.3 mmol/L (3.5-4.9) Serum creatinine 90 µmol/L (60-110) Serum C reactive protein 31 mg/L (<10) Chest x ray shows a moderate right-sided pleural effusion, but no other abnormalities. Bedside pleural ultrasound in the Emergency department shows a confluent anechoic right pleural effusion with no septations and a 3 cm maximal depth. Pleural tap is performed which revealed slightly viscous straw-coloured fluid. Fluid pH is measured 7.18 from a blood gas machine, though laboratory microbiology and biochemistry is awaited. What should be done next? (Please select 1 option) Await pleural fluid Gram stain before any instrumentation Await pleural fluid protein before any instrumentation Large-bore (24F) intercostal drain insertion Pleural aspiration up to 2.5 L maximum Small-bore (12F) intercostal drain insertion

In addition to complicated parapneumonic effusions, a number of chronic pleural processes can cause an effusion with a low pH, namely: rheumatoid arthritis tuberculosis malignancy oesophageal rupture, and lupus pleuritis. This confounds decision making on the need for chest tube drainage. This lady's chronic effusion may be RA-related, therefore there is no compelling argument as of yet for chest-tube drainage, especially given the reasonably favourable (yet non-specific) ultrasound features. Though clearly not empyematous (given the fluid appearance) this effusion would be correctly upgraded to a complicated effusion if the Gram stain revealed organisms, the treatment for which is intercostal drain insertion (not simple aspiration). The size of chest drain required in pleural infection continues to draw debate, however data from the MIST-1 trial of intrapleural streptokinase (a negative study) showed that patients with small-bore drains had comparable outcomes to those with large-bore drains, but better pain scores, therefore small is generally favoured initially. Reference: British Thoracic Society. Pleural Disease Guideline.

A 75-year-old lady with a history of bronchiectasis presents as an emergency with massive haemoptysis and haemodynamic instability. Her Hb has dropped to 70 g/L, platelets are 90 ×109/L and her blood pressure is 100 mmHg systolic. She is tachycardic. You are concerned. You secure intravenous access, administer vitamin K, tranexamic acid and terlipressin bolus. You have also commenced a blood transfusion and broad spectrum antibiotics. Which of the following should be the next step in this case? (Please select 1 option) Administer platelet transfusion Intubate Refer for angiography Refer for bronchial artery embolisation Refer for bronchoscopy

In massive haemoptysis a double lumen endotracheal tube inserted by an experienced anaesthetist can help to tamponade bleeding, protect the non-affected lung and maintain a clear airway, whilst awaiting definitive intervention. In a stable patient presenting with massive haemoptysis there is more time available to localise the bleeding site and determine underlying aetiology. So it would be sensible to send for CT thorax to look for cavities, masses, or vascular lesions. Thereafter bronchoscopy can be conducted to stem the bleeding. If bleeding persists bronchial artery embolisation or surgery may be required. In an unstable patient, the situation can be challenging as the patient may exsanguinate if the bleeding site is not localised quickly. Aggressive resuscitation and notification of the relevant teams (thoracic surgery, respiratory, radiology, intensive care and haematology) is necessary. Early bronchoscopy in the intubated patient is indicated and if the bleeding site is not localised then angiography may be a useful next step. Remember that antifungal cover alongside broad spectrum antibiotics are worth considering in cases of haemoptysis.

A 24-year-old lady with known schizophrenia presents to the acute medical unit with an exacerbation of her asthma. She is able to talk but struggles with full sentences. On examination you note she is thin and complains of breathlessness and a 'tight chest' for 24 hours. Air entry is equal bilaterally, with an expiratory wheeze throughout. Her oxygen saturation on air is 95%, Her respiratory rate is 35, pulse rate 117 bpm. An arterial blood gas (on air) is taken and reveals: pH 7.44 pCO2 5.9 kpa pO2 9.8 kpa HCO3 23.4 BE −1.52 A portable chest x ray reveals hyperexpanded lung fields with no pneumothorax or consolidation. What is the severity of her acute asthma exacerbation? (Please select 1 option) Mild Moderate Acute severe Acute very severe Life threatening

In the context of a raised respiratory rate a patient with acute asthma exacerbation and a normal or raised carbon dioxide level on arterial blood samples should be treated as life threatening. This point is often missed or under-appreciated by assessing physicians and nursing staff. It can imply physical exhaustion is leading to inadequate gas exchange and respiration. In this situation an anaesthetist or ICU doctor should assess the patient for suitability for intubation. Asthma patients who are deteriorating in this way should never be given BIPAP/NIV as a means of reducing pCO2 on a medical unit. They should either be intubated or closely monitored in a suitable high dependency area. In this case, the patient's history of schizophrenia or psychosocial disorder also implies she is at increased risk of death in the context of severe asthma. Reference: British Thoracic Society. Asthma Guidelines.

Question 13 of 31 A 59-year-old lady presents with a very unsteady gait and difficulty in weight bearing which has left her non-ambulatory. The condition has progressed quickly over the past four days following an episode of gastroenteritis. On further questioning the symptoms are consistent with an ascending neuropathy in the lower limbs associated with paraesthesia. A lumbar puncture shows raised protein in the cerebrospinal fluid (CSF). What is the recommended treatment? (Please select 1 option) Hydrocortisone 200 mg iv od Interferon beta Intravenous immunoglobulins (IVIG) Plasmapheresis after treatment with IVIG Prednisolone 80 mg od po

Intravenous immunoglobulins (IVIG) This is the correct answer Intravenous immune globulin (IVIG) is as effective as plasma exchange for the treatment of Guillain-Barré syndrome (GBS). Choice of treatment modality is dependant on local availability (that is, non-specialist centres). Once the mainstay of therapy for GBS, glucocorticoids have not been shown to be beneficial and so no longer have a role. There is no evidence to suggest benefit in using both IVIG and plasmapheresis. Aside from plasma exchange and intravenous immune globulin (IVIG), no other pharmacologic agents have been found to be effective for GBS. Pain is controlled using non-steroidal anti-inflammatory medications (carbamezapine may be used in the intensive care setting). A long period of rehabilitation may follow - around 80% of patients regain baseline neurological function, but up to 10% may not. Reference: Patwa HS, Chaudhry V, Katzberg H, et al. Evidence-based guideline: intravenous immunoglobulin in the treatment of neuromuscular disorders: report of the Therapeutics and Technology Assessment Subcommittee of the American Academy of Neurology. Neurology. 2012;78:1009-15. Hughes RA, Swan AV, van Doorn PA. Intravenous immunoglobulin for Guillain-Barré syndrome. Cochrane Database Syst Rev. 2012;7:CD002063. Kissel JT, Cornblath DR, Mendell JR. Guillain-Barré Syndrome. In: Mendell JR, Kissel JT, Cornblath DR (eds.) Diagnosis and management of peripheral nerve disorders. Oxford University Press: New York; 2001. Hughes RA, Wijdicks EF, Barohn R, et al. Practice parameter: immunotherapy for Guillain-Barré syndrome: report of the Quality Standards Subcommittee of the American Academy of Neurology. Neurology. 2003;61:736-40.

Question 19 of 31 You are called urgently to review a 29-year-old man presenting with seizures. He is known to have epilepsy and is maintained on sodium valproate. He also has a previous history of alcohol abuse. He has been suffering with a seizure for the past 10 minutes. The airway is being managed, and his blood sugar is 6 mmol. Intravenous access has been secured and diazepam has been administered initially by the ambulance crew and again in A&E. Which of the following medications would you choose next? (Please select 1 option) Intravenous lorazepam Intravenous midazolam Intravenous Pabrinex Intravenous phenytoin Intravenous propofol

Intravenous phenytoin This is the correct answer By definition status epilepticus is a seizure which lasts for longer than 30 minutes. However, the longer a seizure lasts the more refractory to treatment it becomes. The average seizure will self terminate after two minutes or so. Emergency management of a seizure is as follows: Airway, breathing, circulation Check for and treat hypoglycaemia and electrolyte disturbance. In status epilepticus: Early management is with intravenous lorazepam or diazepam Established management with intravenous phenytoin, and Refractory management (on intensive care) with propofol, midazolam or thiopentone. Administration of intravenous thiamine may be helpful in those with a history of acute alcohol withdrawal. In dealing with patients please refer to local guidelines as there is little evidence available due to the emergency nature of patient presentation.

Question 21 of 22 A 42-year-old lady presents to the gastroenterology clinic with a six week history of bloating and non-specific abdominal pain. She has recently returned from a trip to India and the symptoms have continued since that time. There has been no documented weight loss but there is a history of change in bowel habit, alternating between toothpaste-like stools to hard pebbles. There has never been any blood or mucous. In which of the following conditions could faecal calprotectin be a useful test? (Please select 1 option) Coeliac disease Colon cancer Giardiasis Irritable bowel syndrome Pseudomembranous colitis

Irritable bowel syndrome This is the correct answer Faecal calprotectin is a protein and has been found in several human biological materials, including: serum saliva cerebrospinal fluid, and urine. In stool faecal calprotectin is found in elevated levels in cases of bowel inflammation, when compared to non-inflamed bowel conditions such as irritable bowel syndrome. An increased calprotectin concentration in stool is the direct consequence of neutrophil degranulation due to mucosal damage, and so it may be used as a marker in response to treatment in inflammatory bowel conditions, for example ulcerative colitis and Crohn's disease. Faecal calprotectin can sometimes generate false positives, in which case endoscopic examination may be indicated. The test has no role if a diagnosis of malignancy is suspected. Reference: NICE. Faecal calprotectin diagnostic tests for inflammatory diseases of the bowel (DG11).

You asked to assess an obese 76-year-old man who has undergone a hemicolectomy for bowel cancer in the past week. He has had a temperature of 38°C since day three post-operatively. His wound is healing well and the operation was documented as uneventful. His other observations are stable and the patient is haemodynamically stable. On examination, he is alert and orientated, with a chest clear, normal heart sounds and he claims that his pain is well controlled. Both legs are grossly oedematous. His surgical team are happy with his abdomen. Investigations reveal: Hb 130 g/L WCC 11.1 ×109/L Platelets 389 ×109/L Renal function normal CRP 80 mg/L (down from 103 mg/L) Urine dip normal CXR normal Blood cultures negative to date Which of the following represents the most likely diagnosis in this? (Please select 1 option) Deep vein thrombosis Infective endocarditis Intra-abdominal abscess Surgical site infection Ventilator associated pneumonia

It is quite common for a patient to have a low grade fever for the first few days following a major operation. The large amount of tissue damage triggers pro-inflammatory cytokine release and a systemic inflammatory response. However a persistent fever may signal a serious underlying pathology. There is no obvious source of infection in this case despite the patient's persistent fever. He has multiple risk factors for venous thrombus including immobility, obesity and major surgery.

A 62-year-old man with a history of type 2 diabetes, hypertension and ischaemic heart disease presented to the emergency department with an episode of LVF for the third time in the past four months. He responded well to a bolus of furosemide and a period of CPAP and you review him on the post take ward round. Medication includes lisinopril, bisoprolol, furosemide, doxazosin, aspirin, metformin and sitagliptin. Examination reveals a BP of 145/72 mmHg, pulse is 80 and regular. He is not in left ventricular failure. Investigations reveal: Hb 135 g/L (135-180) WCC 6.3 ×109/L (4.5-10) PLT 175 ×109/L (150-450) Na 137 mmol/L (135-145) K 4.8 mmol/L (3.5-5.5) Cr 121 µmol/L (70-110) ECG shows no acute changes. CXR shows LVF on admission film. Which of the following medications would you reduce or discontinue given this episode of heart failure? (Please select 1 option) Bisoprolol Doxazosin Lisinopril Metformin Sitagliptin

It was the ALLHAT study which first raised questions around use of alpha blockers and increased risk of left ventricular failure where, versus other antihypertensive options, a doubling of heart failure risk was seen for patients treated with doxazosin. In this patient, who is optimally treated apart from the alpha blocker, discontinuation may reduce the risk of future presentations with acute LVF. Whilst beta blockade may precipitate acute heart failure, use of cardioselective beta blockers such as carvedilol or bisoprolol is recommended because of their positive effect on outcomes including mortality. As such, given there are no residual signs of fluid overload, the bisoprolol should be continued. Neither metformin or sitagliptin are thought to contribute to risk of LVF, although saxagliptin, (a drug in the same class as sitagliptin) has been found to be associated with increased risk of cardiac failure in a recently published cardiac outcome study. Reference: Messerli FH. Doxazosin and congestive heart failure. J Am Coll Cardiol. 2001;38(5):1295-6.

Question 17 of 31 A 30-year-old male is complaining of night blindness and loss of peripheral vision. Upon examination you also notice upon that he has limitation of extraocular eye movements in all directions of gaze, but does not complain of diplopia. He has a pacemaker fitted. Upon fundoscopy you notice the following changes: What is the likely diagnosis? (Please select 1 option) Bardet-Biedl syndrome Bassen-Kornzweig syndrome Kearns-Sayre syndrome Laurence-Moon syndrome Usher syndrome

Kearns-Sayre syndrome This is the correct answer The fundoscopy image shows bone spicule-like pigmentary changes within the mid-periphery of the retina with attenuated vessels and waxy disc pallor. These fundal appearances are consistent with that of retinitis pigmentosa. There are a number syndromes which are associated with this condition. The combination of chronic external ophthalmoplegia (CEO) which accounts for the eye movement signs, heart conduction defects (pacemaker) and the fact he is male (mitochondrial inheritance) make the diagnosis of Kearns-Sayre disease highly probable in the case described. All of the conditions mentioned above are associated with a pigmentary retinopathy consistent with retinitis pigmentosa (RP): Bardet-Biedl - RP associated with obesity, polydactyly and hypogenitalism Bassen-Kornzweig - RP with associated with abetalipoproteinaemia, acanthocytosis, ataxia Laurence-Moon - RP with associated hypogenitalism, mental retardation, spastic paraplegia Usher - RP with associated sensor neuronal deafness, ataxia and neurological abnormalities. The numerous syndromes associated with retinitis pigmentosa are a favourite for examiners. It can be useful to memorise the triad of bone spiculated mid-periphery pigmentation, attenuated vessels and waxy disc pallor for retinitis pigmentosa.

Question 9 of 22 Core Questions A 21-year-old physiotherapy student from Canterbury has been referred by the GP to the medical department for dyspepsia, which is not resolving with antacids and a trial of proton pump inhibitor (PPI). He is otherwise fit and well, he plays sports, does not smoke or consume alcohol and has no family history of cancer. An endoscopy is conducted to exclude peptic ulceration. The endoscopy is normal, however the Helicobacter pylori serology test has come back positive. Which of the following would be the appropriate course of action? (Please select 1 option) Likely false positive, ignore this result Likely false positive, re-scope and biopsy for H. pylori in one month Likely false positive, re-test using breath test or stool test Likely true positive, commence triple therapy Likely true positive, prescribe long term PPI therapy

Likely false positive, re-test using breath test or stool test This is the correct answer This is a young man with a very low pre-test probability for H. pylori, with no evidence of peptic ulceration and he is from a region in the world where prevalence of H. pylori is low. Therefore this is very likely to be a false positive. Rather than to subject this 21-year-old to a course of unnecessary treatment it would be sensible to re-test using either urease breath test or a faecal assay. Histology via endoscopy is of course more accurate, however you would not subject a patient to endoscopy simply to test for H. pylori; this approach is only used when the patient is already undergoing endoscopy for another reason. If this patient were to have peptic ulceration or a high pre-test probability for H. pylori infection, it would be reasonable to go ahead and initiate treatment. Serological conversion from positive to negative may be monitored in such cases to ensure that the infection has been successfully eradicated.

A 68-year-old woman presents to the Emergency department with sudden onset palpitations. She had recently seen her GP with a productive cough and had been put on a course of antibiotics. Past medical history consists of atrial fibrillation, hypertension, type 2 diabetes mellitus and anxiety. Drug history consists of warfarin 3 mg, digoxin 125 mcg, bisoprolol 2.5 mg, gliclazide 40 mg, diazepam 5 mg and erythromycin 500 mg. An ECG shows sinus rhythm 80 bpm with a QTc of 500 milliseconds. Which of her above medications is likely to be the cause of the prolonged QTc? (Please select 1 option) Bisoprolol Diazepam Digoxin Erythromycin Gliclazide

Long QT syndrome is associated with the potentially deadly rhythms torsades de pointes and ventricular fibrillation. There are over 50 common prescribed medications known to prolong the QT interval. Antiarrhythmics are the most potent. These include common medications such as amiodarone and sotalol. Antidepressants (tricyclics and SSRIs) and antipsychotics (haloperidol and phenothiazides) have been well documented to prolong the QT interval. Other common classes of drugs that people with long QT syndrome should avoid include antibiotics such as macrolides and quinolones, as well as antiemetics such as ondansetron. Further Reading: NCBI: Drug-induced QT interval prolongation and torsades de pointes SADS (Sudden Arrhythmic Death Syndrome). Drugs to avoid.

Question 13 of 15 A 50-year-old female Afro-Caribbean patient is admitted with allergic angio-oedema. Which of the following medications is most likely to have caused this presentation? (Please select 1 option) Codeine phosphate Cromoglicic acid Loratadine Losartan Nitrofurantoin

Losartan This is the correct answer Allergic angio-odema can be triggered by certain foods and drugs. Commonly recognised precipitants include angiotensin converting enzyme (ACE) inhibitors and non-steroid anti-inflammatory drugs. Black people being treated with ACE inhibitors are three times more likely to suffer this side effect compared with Caucasian people. Commonly used drugs but less well known to cause angio-oedema include: angiotensin 2 receptor blockers (ARB) proton pump inhibitors (PPI), and selective serotonin reuptake inhibitors (SSRI). Reference: NHS Choices. Causes of angioedema.

A 43-year-old businessman from Saudi Arabia presents unwell with a four day history of cough and shortness of breath. He has a past medical history of diabetes, renal impairment and obesity. He has been taking amoxicillin and clarithromycin orally, prescribed by a doctor, but despite this his symptoms have been deteriorating. You request a chest x ray which shows patchy consolidation in the right upper lobe. An arterial blood gas is as follows on 15 L oxygen: pH 7.31 PaO2 8.1 PCO2 3.1 On examination he is distressed, with HR 130/min, sinus tachycardia, BP 90/50 mmHg, and temperature 38.4°C. Blood tests reveal: WCC 9 ×109/L Hb 135 g/L Plts 470 ×109/L Na 131 mmol/L K 4.8 mmol/L Urea 14 mmol/L Creat 138 μmol/L Which of the following diagnoses is most likely? (Please select 1 option) Massive pulmonary embolism (PE) Middle East respiratory syndrome (MERS) Mycobacterium tuberculosis (TB) Mycoplasma pneumonia Pneumococcal pneumonia

MERS is viral respiratory illness first reported in Saudi Arabia in 2012. It is caused by a coronavirus. It is more likely in patients who are overweight, have diabetes or end stage renal failure. MERS spreads quickly through droplet infection and a 'super spreader' on a plane can infect several passengers. As a result international travel by flight is a prime medium for infection. Patients deteriorate quickly and there is a high mortality rate. As it is a virus, antibiotics typically show no improvement. Chest x ray may show changes in the lung parenchyma, though a white cell count may well be normal at presentation. Management of these patients is supportive; intubation and ventilation in the ITU setting is usually required. If a case of MERS is encountered in hospital it should be reported.

Question 18 of 31 A 23-year-old female experiences a sharp pain along the left side of her neck. She had recently been to the hairdressers. On examination you notice a mild ptosis and miosis to the left eye. What is your immediate management of this patient? (Please select 1 option) 4% cocaine test Anti-migraine medication Chest radiograph MRI/A of head MRI/A of the head, neck and top of thorax

MRI/A of the head, neck and top of thorax Correct The symptoms and signs are consistent with that of a likely diagnosis of acute Horner's syndrome secondary to dissection of the internal carotid artery. Urgent imaging of the sympathetic tract must be undertaken. This includes imaging from the top of the thorax to the neck and of the cavernous sinuses in the brain. A 4% cocaine test confirms Horner's syndrome, but it can take up to one hour and it is rarely done in clinical practice. The above presentation is a medical emergency and requires immediate diagnosis and intervention. Migraines are a cause of Horner's syndrome but given the history there is no evidence of headache, so anti-migraine medication would be incorrect in this instance. A chest radiograph would look for a Pancoast tumour for a cause of Horner's, which is unlikely in this case. An MRI/A of head will show up third order causes of a Horner syndrome, in particular in the cavernous sinus.

Question 4 of 31 Core Questions A previously healthy 38-year-old woman of Bangladeshi origin presented to the Emergency department with a three day history of flu-like symptoms, left sided headache and blurred vision. Further questioning revealed that three months earlier she had suffered an episode of ataxia and lower limb numbness, resolving spontaneously after three weeks. MRI lumbar spine had shown only left sided scoliosis at L2 with minor degenerative changes. There was no history or signs of meningism or exanthematous disease, and the patient reported no recent vaccinations or new medications. There was no similar past or family history. Examination of visual acuity was extremely poor - grossly finger counting and revealed slight right eye relative afferent pupillary defect (suggestive of optic neuritis) and bitemporal pallor on fundoscopy. Neurological examination revealed an unsteady gait with a tendency to fall forwards, which was attributed to the patient's poor vision. Tone, power, sensation and co-ordination were normal in all four limbs, as were reflexes and proprioception. CT brain failed to show any findings. MRI scans are shown below with abnormalities highlighted: The CSF showed raised protein but was negative for oligoclonal bands. What is your diagnosis? (Please select 1 option) Metastatic lesions Multiple sclerosis Neuromyelitis optica Tuberculosis Vasculitis

Neuromyelitis optica This is the correct answer When faced with a presentation of sudden onset, rapidly worsening vision in an Asian patient with normal MRI brain, the possibility of Devic's disease should be borne in mind. Devic's is a variant of MS and presents with spinal demyelination and absent oligoclonal bands. The pattern of events here suggested a clinical diagnosis of acute demyelination with deteriorating vision. The patient was treated with IV methylprednisolone followed by oral steroids, resulting in complete remission of symptoms. Devic's disease, also known as neuromyelitis optica (NMO), is defined as optic neuritis with myelitis. The main clue to diagnosis here was the onset of bilateral reduction in visual acuity. Here is a useful aide for differential diagnosis in rapid onset blindness: Devic's Disease Multiple Sclerosis Asian communities Western communities Male:female ratio equal Female preponderance Negative brain MRI at onset MRI brain: plaques/demyelination Oligoclonal bands positive 30% Oligoclonal bands positive 80% Unilateral or bilateral optic neuritis Usually unilateral optic neuritis MRI abnormality >3 vertebral segments Reference: Devic.org.uk. What is Devic's Disease? Kumar P, Clark M. Kumar and Clark Clinical Medicine. 4th edn. London: Saunders; 1998. pp 1015-1016. Mukherjee E. Devic's Disease or Multiple Sclerosis? The West London Medical Journal 2009;1:31-34. The Transverse Myelitis Association. Neuromyelitis Optica.

A 56-year-old man has been admitted to the coronary care unit after suffering an anterior myocardial infarction. The nurse calls you to review him as his trace is changing on the monitor intermittently. His observations have remained stable and he denies any symptoms other than a little light-headedness. He has been commenced on a few new medications, including bisoprolol, ivabradine, lisinopril, aspirin, clopidogrel and atorvastatin. The nurse has taken a 12 lead trace of the worrying change in heart rhythm which she shows you: Which of the following would be the most suitable line of management for this patient? (Please select 1 option) Observe Refer for implantable cardioverter defibrillator (ICD) Stop the beta blocker Temporary pacing wire Trial of atropine

New-onset third-degree atrioventricular (AV) block (that is, complete heart block) is a medical emergency. Treatment can be gauged on presentation and history. Recent MI, in particular anterior wall, indicates AV ischaemia and possibility of asystole. However, treatment with new AV node blocking drugs may also result in complete heart block and a trial of cessation would be warranted if the clinical scenario allowed. Beyond this management should be focussed on initiating pacing of some kind. The symptomatic or high-risk patient may require immediate intervention with transcutaneous or temporary wire pacing, whilst the more stable asymptomatic patient is able to be observed whilst awaiting a permanent pacemaker or ICD insertion.

An 82-year-old female with severe COPD is admitted with a three day history of worsening cough, yellow sputum production and wheeze. She has no LTOT or home nebulisers. Her comorbidities include type 2 diabetes mellitus, congestive cardiac failure, paroxysmal atrial fibrillation and rheumatoid arthritis. Her baseline exercise tolerance is 10 yards with a Zimmer frame. She has an advanced directive stating her desire never to be resuscitated, nor intubated and ventilated. After five hours of management in the Emergency department, including 2 × 2.5 mg salbutamol nebulisers, IV hydrocortisone 100 mg, and oral doxycycline, her arterial blood gas (ABG) on FiO2 0.24 is as follows: PO2 7.8 kPa (11.3-12.6) PCO2 7.5 kPa (4.7-6.0) pH 7.27 (7.35-7.45) Bicarbonate 35 mmol/L (21-29) SaO2 88% (94-98) What is the next most appropriate intervention, prior to repeating the ABG? (Please select 1 option) Aminophylline infusion BiPAP, starting at 12/4 cmH2O and titrate pressures according to repeat ABG Increase FiO2 to 0.28 Magnesium sulphate infusion Repeat salbutamol/add ipratropium nebulisers

Non-invasive ventilation (NIV) is indicated for acute hypercapnic respiratory failure, but only after 'maximal' medical treatment. Therefore, it is prudent to consider further nebulised β-agonists and to trial anti-cholinergics, as this may avoid the need for NIV. Aminophylline offers bronchodilation via a different mechanism (through non-selective phosphodiesterase inhibition) though controlled studies have failed to demonstrate clear benefits. In addition to having a narrow therapeutic index, it is commonly pro-arrhythmic, and is often best avoided in patients with co-existent severe cardiac disease. Magnesium sulphate, though of proven benefit in acute asthma, has not been shown to be beneficial in acute COPD. Whilst likely hypoxic compared to her baseline, target oxygen saturations should be maintained at 88-92%, therefore the FiO2 is satisfactory. Reference: British Thoracic Society. Emergency Oxygen Use in Adult Patients Guideline. British Thoracic Society Standards of Care Committee. Non-invasive ventilation in acute respiratory failure. Thorax. 2002;57:192-211.

Question 8 of 22 Core Questions A 33-year-old male health care assistant presents with a flare of symptoms. He is known to suffer from Crohn's disease. He is complaining of abdominal pain and bowel motions approximately three to four times per day, there is no vomiting. Observations show a HR of 88/min, temperature 36.8°C, saturation 97% air, BP 110/75 mmHg. An abdominal x ray excludes obstruction. Which of the following is most appropriate? (Please select 1 option) Admit the patient for intravenous fluids, glucocorticoids and antibiotics Admit the patient for intravenous glucocorticoids Commence azathioprine and discharge Commence olsalazine and discharge None of the above

None of the above This is the correct answer This is a case of mild to moderately severe Crohn's disease. Some of the markers of severity include: abdominal tenderness presence of a mass or obstruction in the abdomen toxicity, and weight loss. The mainstay of treatment for acute Crohn's disease is with glucocorticoid therapy. Mild to moderately active Crohn's may be treated with oral prednisolone or controlled ileal release (CIR) budesonide in the outpatient setting. One of the key questions to ask is if the patient is able to tolerate oral fluids and diet. This will ensure that oral glucocorticoid treatment will be effective and that the patient will not become dehydrated. With Crohn's disease strictures, mass, abscess and intestinal obstruction all need to be excluded early, so imaging the bowel with CT if any of these are suspected is important. The evidence for using 5-aminosalicylic agents and sulfasalazine is mixed, though there is no harm in increasing the dose of pre-existing 5-ASA drugs during a flare. Antibiotics are warranted in those presenting with signs of systemic toxicity, broad spectrum antibiotics such as ciprofloxacin and metronidazole would be of choice. Budesonide has been found to be effective for patients who are unable to tolerate glucocorticoid therapy due to systemic side effects. Azathioprine and mercaptopurine are often used as monotherapy to maintain remission. Methotrexate may be used in those with thiopurine s-methlytransferase (TPMT) deficiency. In patients taking methotrexate as a regular therapy, it is important to look for neutropenia at presentation. Reference: NICE. Crohn's disease: management (CG152).

Question 18 of 22 A 22-year-old student presents to the acute medicine ambulatory care service with a history of malaise for the last four months. There is a past history of IV drug abuse. His GP has checked some liver function tests and viral hepatitis serology and wants the results interpreted. Investigations reveal: Hb 124 g/L (135-180) WCC 8.1 ×109/L (4.5-10) PLT 330 ×109/L (150-450) Na 138 mmol/L (135-145) K 4.9 mmol/L (3.5-5.5) Cr 110 µmol/L (70-110) AST 102 IU/L (5-35) Alkaline phosphatase 125 U/L (35-100) Bilirubin 18 µmol/L (<26) HBsAg Positive Anti-HBc Positive IgM anti-HBc Negative Anti-HBs Negative Which of the following is the most likely diagnosis? (Please select 1 option) Acute hepatitis B infection Chronic hepatitis B infection Immunity due to cleared infection Immunity due to vaccination Resolving acute hepatitis B infection

OGD and colonoscopy This is the correct answer Iron deficiency anaemia is a red flag sign and needs to be managed appropriately. A good history and examination, followed by investigation with haematinics, iron levels, blood film, and screening for specific conditions is required. If a patient presents with a microcytic anaemia and low ferritin, the next reasonable step is to exclude coeliac disease. In this case coeliac has successfully been excluded. The patient is premenopausal with no actual upper GI symptoms; the next step as per British Society of Gastroenterology (BSG) guidance is to book for an OGD and colonoscopy. If the coeliac screen comes back as positive, it would be pertinent to confirm the diagnosis with duodenal biopsies. Reference: Goddard AF, James MW, McIntyre AS, et al. Guidelines for the management of iron deficiency anaemia. Gut. 2011;60:1309-16.

A 32-year-old city trader is brought to the Emergency department with central crushing chest pain. He was apparently dancing with friends at a club when he had to sit down because of feeling faint. By the time he arrives in the department he is pale and covered in sweat. He smokes 20 cigarettes per day and is a heavy cocaine user. Examination reveals a BP of 165/100 mmHg, pulse is 105 and regular. There are bilateral basal crackles on auscultation consistent with left ventricular failure. Investigations reveal: Hb 140 g/L (135-180) WCC 5.1 ×109/L (4.5-10) PLT 381 ×109/L (150-450) Na 138 mmol/L (135-145) K 3.6 mmol/L (3.5-5.5) Cr 105 µmol/L (70-110) ECG shows anterior ST elevation. CXR shows LVF on admission film. He is given bivalirudin, aspirin and clopidogrel and prepared for possible percutaneous intervention. What other step would you take? (Please select 1 option) CPAP IV beta blocker IV glycoprotein IIb IIIa inhibitor IV nitrate Oral calcium channel antagonist

One possible explanation here is coronary artery vasospasm as a result of excessive cocaine use. The alternative is that the patient is suffering an acute STEMI related to accelerated coronary artery disease. In this situation nitrate is the best option because it will work as a treatment for vasospasm and also reduce pre-load and improve the symptoms of LVF. Glycoprotein IIb IIIa antagonists are not recommended when rapid progression to the cardiac catheter suite is a possibility and should therefore be avoided here. Beta blockade may worsen ischaemia in this situation and is therefore also not recommended. In the absence of significant LVF, calcium antagonists can be used as a first line alternative to nitrates. CPAP is not recommended in LVF related to acute ischaemia where other therapeutic options are available.

Question 7 of 22 Core Questions A 38-year-old man is admitted with a history of intermittent diarrhoea. Upon further questioning he mentions episodes of flushing and wheeze. He has no past medical history. You suspect that he may be suffering from carcinoid syndrome and auscultate the cardiac region. If this is indeed a diagnosis of carcinoid syndrome, which of the following murmurs is most likely to be present? (Please select 1 option) Ejection systolic murmur Mid-diastolic murmur Pansystolic murmur loudest at the apex radiating to the axilla Pansystolic murmur loudest at the left sternal edge Systolic murmur in the pulmonary region

Pansystolic murmur loudest at the left sternal edge This is the correct answer Heart murmurs can arise in carcinoid as a consequence of plaque-like thickening on valvular cusps. Fifty percent of patients with carcinoid syndrome have cardiac involvement which is predominantly right-sided. Tricuspid regurgitation is the most common valvular lesion. Carcinoid is a neuroendocrine tumour which paroxysmally releases hormones, such as serotonin which may result in: flushing diarrhoea abdominal cramping peripheral oedema, and wheeze. Commonly there is association with intake of alcohol or coffee triggering a paroxysm. Surgery may be curative in an isolated lesion, but there is no effective cure in case of metastasis. Somatostatin anologues have been used to decrease release of secretory hormones in such cases.

You are sending home a patient who has been successfully treated by intercostal drainage for a first spontaneous pneumothorax. He tells you he is planning on flying to Cuba and then on to a resort in Varadero next month where he hopes to do some diving. What advice should you give him? (Please select 1 option) He can fly after six weeks and dive after 3 months He can fly 1 week after full resolution but cannot dive He can fly and dive but must wait three months He cannot dive ever again but can fly immediately He cannot fly and cannot dive ever again

Patients may travel safely one week after full resolution on chest x ray, otherwise, there is still a significant risk of recurrence for up to one year, depending on whether the patient has underlying lung disease or not. Many commercial airlines previously advised arbitrarily a 6-week interval between the pneumothorax event and air travel, but this has since been amended to a period of 1 week after full resolution. After a pneumothorax, diving should be discouraged permanently unless a very secure definitive prevention strategy such as surgical pleurectomy has been performed. Reference: MacDuff A, Arnold A, Harvey J. Management of spontaneous pneumothorax: British Thoracic Society Pleural Disease Guideline 2010. Thorax. 2010;65:ii18-31.

A 65-year-old man is admitted with sudden onset of left sided chest pain that began 12 hours previously. On examination, he is asymptomatic with a respiratory rate of 14/min and oxygen saturations of 94% on room air. There is hyperresonance to percussion over the left hemithorax. He has no past medical history of note and takes no regular medication, but has a 60 pack year smoking history. The admission chest x ray is shown. What is the next step? (Please select 1 option) Admit, administer high-flow oxygen, and observe Admit, and book CT-guided aspiration Aspirate with 16-18G cannula Large bore (24F) intercostal drain insertion Small bore (12F) intercostal drain insertion

Patients over 50 with a significant smoking history are likely to have underlying structural lung disease and as such should be managed as a secondary spontaneous pneumothorax (SST), inferring that they are less likely to benefit from needle aspiration alone. However in the 2010 update to the BTS pleural disease guidelines, a subcategory was added to the SST side of the algorithm, allowing the option for needle aspiration of small pneumothoraces of 1-2 cm with minimal symptoms, reflecting the morbidity associated with intercostal drains. In simple, non-traumatic pneumothoraces, large-bore drains (>20F) offer no benefit over small-bore drains (<14F), and CT-guidance is not required. Size estimation should be done at the hilar rim (point b): Reference: British Throacic Society. Pleural Disease Guideline. MacDuff A, Arnold A, Harvey J. Management of spontaneous pneumothorax: British Thoracic Society Pleural Disease Guideline 2010. Thorax. 2010;65:ii18-ii31.

A 72-year-old woman comes to the Emergency department ten days after discharge following stenting of the LAD and the mid-right coronary artery for critical stenoses. She complains of severe left sided and central pleuritic chest pain and increased shortness of breath. Examination reveals a BP of 132/80 mmHg, pulse is 80 and regular, JVP is slightly elevated. There is no pericardial rub although heart sounds appear quiet. Investigations reveal: Hb 120 g/L (115-160) WCC 10.3 ×109/L (4-10) PLT 280 ×109/L (130-400) Na 137 mmol/L (135-145) K 4.7 mmol/L (3.5-5.5) Cr 121 µmol/L (50-90) CRP 110 (<10) Troponin <0.05 - ECG show widespread T wave inversion. What is the most likely diagnosis? (Please select 1 option) Acute pericarditis Cardiac tamponade Lower respiratory tract infection NSTEMI Pulmonary embolus

Pericarditis post angioplasty and stenting is well described, and can occur any time between 48 hours and two weeks after the procedure. The symptoms including pleuritic chest pain, elevated inflammatory markers, and non-specific ECG changes fit best with this diagnosis. Echocardiography is indicated to confirm the size of any pericardial effusion and there is no significant haemodynamic compromise. Treatment options include NSAIDs, colchicine and prednisolone. The lack of significant haemodynamic compromise effectively rules out cardiac tamponade, and the normal troponin rules out NSTEMI. Whilst arterial rupture and consequent haemopericardium is seen, symptoms develop peri-procedure. Pulmonary embolus may be a differential, but given patients undergoing PCI are mobilised quickly after procedure, and anti-coagulated, it is unlikely. Further Reading: Shah NH, Scriven A. Post Cardiac Injury Syndrome after Uncomplicated Coronary Angioplasty. J Invasive Cardiol. 2009;21:E16-E17.

Question 15 of 31 A 10-year-old child presents with pyrexia with swollen upper and lower eyelids. His left eye is red with chemosis. On examination his vision is reduced to 6/24 on a Snellen chart. There is limitation of extra ocular movements in all gazes and proptosis. His mother explains that he has been suffering from sinusitis recently. You request an urgent CT scan of his orbits and head. What should you look for in the CT scan? (Please select 1 option) Capillary haemangioma of the orbit Enlarged lacrimal gland Neurofibroma Periosteal abscess Rhabdomyosarcoma

Periosteal abscess This is the correct answer These symptoms and signs are consistent with that of a likely diagnosis of orbital cellulitis. Key signs and symptoms include pyrexia and proptosis which are limiting his extraocular movements. Reduction of his vision indicates there may be compression of the optic nerve. This is an ophthalmic emergency and has associated mortality if not treated promptly. It is important to ask for any history of sinusitis since the organisms may have originated from the sinuses (which is likely in this case). Periosteal abscess is a known complication of this condition and so ENT would need to be contacted for subsequent management. Capillary haemangioma of the orbit could give rise to orbital swelling, however it is not infective in nature and is likely to have been present from birth. Enlarged lacrimal gland would only cause swelling in the upper eyelid and is unlikely give rise to optic nerve compression. Additionally, it would not be septic unless it was infected. Neurofibroma can give rise to proptosis but no upper and lower lid swelling. Rhabdomyosarcoma is known to mimic orbital cellulitis. However patients are not septic and there may be other signs of malignancy.

A 24-year-old man is seen in the acute medical unit (AMU) with palpatations and and abnormal ECG, having been referred by his GP. His investigations are as follows: Hb 154 g/L (130-180) WCC 8.7 ×109/L (4-11) Platelets 184 ×109/L (150-400) CRP 2 (<5) Trop I <0.1 (<0.1) Serum sodium 140 mmol/L (137-144) Serum potassium 4.8 mmol/L (3.5-4.9) Serum urea 7.2 mmol/L (2.5-7.5) Serum creatinine 95 µmol/L (60-110) Chest x ray showed normal heart size and clear lungs. ECG showed sinus rhythm 95 BPM, pre-excitation with a shortened PR interval and widened QRS complex with a delta wave. What is the most appropriate next investigation or treatment in this patient? (Please select 1 option) Amiodarone Echocardiography Empiric treatment beta blockers EP ablation Nil treatment

The most common pre-excitation syndrome is Wolff-Parkinson-White (WPW) . It is associated with an abnormal accessory pathway from the atria to the ventricles and is commonly present with supraventricular tachycardia (SVT). It is a potentially serious condition with a small percentage (<1%) at risk of sudden cardiac death (SCD). The incidence of congenital heart disease has been documented to be as high as 30% in patients who present with pre-excitation. Therefore, an echocardiogram should be performed to rule this out as a contributing factor. Ebstein's anomaly (apical displacement of the septal and posterior tricuspid valve leaflets, leading to part of the right ventricle becoming part of the right atrium) is the most common associated congenital heart disease condition. Mitral valve prolapse and hypertrophic cardiomyopathy have also been documented. Exercise testing is also worthwhile in providing more information on risk assessment of sudden death. Evidence of the delta wave disappearing on exercise has been reported to coincide with lower risk of SCD. Reference: Medscape. Wolff-Parkinson-White Syndrome. Patient.info. Wolff-Parkinson-White Syndrome.

Question 2 of 22 A 19-year-old lady is admitted to hospital following collapse. She is also severely dehydrated. The patient is known to suffer from anorexia and her body mass index is 14 kg/m2 (18-25). She has not been eating solid food for over a month now. She is admitted to hospital for rehydration and feeding. Which of the following should be very closely monitored during this time? (Please select 1 option) Calcium Glucose Phosphate Sodium Zinc

Phosphate Correct Refeeding syndrome is well recognised in severely malnourished patients. Cellular uptake of potassium, phosphate and magnesium can result in precipitous drops in these electrolyte levels, resulting in complications including cardiac arrhythmias. The main electrolytes which ought to be monitored during refeeding syndrome are phosphate, potassium and magnesium. Thiamine supplementation is recommended during the first 10 days of refeeding. The calorific intake is gradually titrated up over time. Zinc deficiency is recognised in long term total parenteral nutrition, and this may present with an inflammatory rash around the mouth or anus, known as acrodermatitis enterohepathica.

A 66-year-old patient with a usual exercise tolerance of 300 yards and an established diagnosis of idiopathic pulmonary fibrosis (IPF) is recovering from an infective exacerbation on the acute medical unit. Whilst reviewing his notes you note his spirometry: FEV1 1.8 litres (57% predicted) FVC 2.2 litres (55% predicted) Ratio 0.81 He asks you whether there is any treatment that could help slow down his progression. From which of the following may he benefit? (Please select 1 option) Azathioprine Long term prednisolone 40 mg with bisphosphonate and acid suppression Pirfenidone Pulsed intravenous methylprednisolone Referral to palliative care for admission to hospice

Pirfenidone is recommended by NICE for those with IPF and an FEV between 50-80% predicted, provided the manufacturer agrees a discounted rate for the drug. It may help to slow progression of the disease. Pulsed IV methylprednisolone is reserved for acutely progressive illness which is not the case here as the patient is actually recovering from a recent illness. There is no evidence for prednisolone alone in IPF and in fact it may increase mortality. Azathioprine should used in conjunction with N-acetylcysteine (NAC) and steroid was suggested as triple therapy for IPF but has since been discredited by the PANTHER trial due to serious side effects. The fact he still walks 300 yards suggests he is not ready for a hospice yet, although palliative care in the future may be reasonable given his underlying progressive disease. Reference: Bradley B, Branley HM, Egan JJ, et al. Interstitial lung disease guideline: the British Thoracic Society in collaboration with the Thoracic Society of Australia and New Zealand and the Irish Thoracic Society. Thorax. 2008;63:v1-58. NICE. Pirfenidone for treating idiopathic pulmonary fibrosis (TA282).

Question 14 of 31 A 28-year-old Polish lady presents to you with her husband. She is a full time housewife with two children. She is upset as she describes an episode of transient loss of vision followed by collapse. Now she has regained consciousness and it appears that she has significant gaps in her memory. She is still complaining of blurred vision, is tearful, feels isolated and wants to go back to Poland to be with her family. Her husband is very quiet and you sense some tension between them. There is little to find on her neurological exam, except subjective claims of blurred vision. All of her blood tests including autoimmune screen and ESR are normal. A CT brain has been reported as normal and CSF results are as follows: Protein 0.66 mmol/L Glucose 2.7 mmol/L (BM 5.6) WCC 2 cells/mm3 RBC 4000 cells/mm3 (bottle 1, decreasing RBCs in subsequent bottles) Gram stain negative Her brain MRI is as follows: Which of the following is the most likely diagnosis given her findings? (Please select 1 option) Posterior circulation stroke Psychological Subarachnoid haemorrhage Temporal arteritis Vasculitis

Posterior circulation stroke This is the correct answer The MRI findings confirm the diagnosis here. Psychological cause for neurological presentation can be only be diagnosed after all other organic lesions have been investigated. Therefore it can only be a diagnosis of exclusion. All possible avenues must be excluded before arriving at this diagnosis. The MRI shows high signal in the posterior circulation. This lady had suffered an embolic phenomenon and was transferred to a stroke unit.

Question 12 of 22 A 45-year-old homeless man is admitted on Christmas day. He was found slumped outside a youth hostel. He is presently disoriented with a Glasgow coma scale of 12 (E3, V 4, M5). Blood tests reveal that he has developed a symptomatic hepatic failure, there is little history available but you suspect there may be alcohol abuse involved. Liver function tests are as follows: Bilirubin 110 µmol/L Alkaline phosphatase 130 U/L Alanine transaminase 200 IU/L Aspartase transaminase 194 IU/L INR 1.6 Albumin 32 g/L Creatinine 101 mmol/L Urea 7.8 mmol/L His Maddrey's discriminant factor is calculated at 25. Which of the following are correct with regard to use of Maddrey's discriminant factor score? (Please select 1 option) Predicts 90 day mortality in alcoholic hepatitis Predicts benefit of use of steroids in acute liver failure Predicts disease severity and mortality risk in alcoholic hepatitis Predicts if a patient with hepatitis is responding to glucocorticoid therapy Predicts mortality in alcoholic hepatitis

Predicts disease severity and mortality risk in alcoholic hepatitis This is the correct answer Patients with a Maddrey's discriminant factor value ≥32 have high short term mortality and may benefit from treatment with glucocorticoids. Patients with lower scores have low short term mortality and do not appear to benefit from glucocorticoids. Use of glucocorticoids is only advised once acute infection or infection with hepatitis B + C have been excluded. This patient is likely to be suffering from moderate alcoholic hepatitis. In addition to Maddrey's discriminant factor there are a range of scoring systems available to calculate his disease severity and mortality rate. Model of end stage liver disease score (MELD) is a statistical model developed to predict survival in patients with cirrhosis. This score has also been used to predict mortality in patients hospitalised for alcoholic hepatitis. Glasgow alcoholic hepatitis score (GAH) predicts mortality in alcoholic hepatitis and is calculated using age, serum bilirubin, renal function, prothrombin time, and peripheral white blood cell count. A GAH score ≥9 had lower sensitivity but higher specificity for predicting 28 day mortality than a Maddrey's discriminant factor of ≥32. Similar results have been found with a MELD score >11. The Lille score is one way of determining if a patient with alcoholic hepatitis is responding to treatment with glucocorticoid therapy. Reference: Forrest EH, Evans CD, Stewart S, et al. Analysis of factors predictive of mortality in alcoholic hepatitis and derivation and validation of the Glasgow alcoholic hepatitis score. Gut. 2005;54:1174-9.

Question 25 of 31 A 60-year-old male presents with blurry vision for two months to his left eye. He is a known type 2 diabetic. Upon ophthalmoscopy you notice the following changes below: What would be your subsequent management for this patient? (Please select 1 option) Advise on commencing insulin Explain that blurry vision is due to cataracts Observe only Referral to the Diabetic Screening Service Referral to the Hospital Eye Service (HES)

Referral to the Hospital Eye Service (HES) Correct The fundal picture show changes consistent with that of diabetic maculopathy. There are hard exudates and haemorrhages and likely macular oedema (this will not be apparent with a direct ophthalmoscope). This patient needs to be referred to the Hospital Eye Service (HES). Treatments now approved by NICE include intravitreal therapy and steroid implants as opposed to the traditional macular laser treatment which is also available to the patient. Good blood glucose control is important for reduction of diabetic retinopathy in the long term but does not address the acute eye condition in this case. Diabetic eye changes are also known to worsen on commencing insulin. The blurry vision on this occasion is not due to cataracts, as outlined above. Diabetic maculopathy is one of the major causes of visual loss and prompt access to treatment must be sought, so observation would be inappropriate in this case. Likewise, referral to the Diabetic Screening Service will only delay assessment, adequate diagnosis and management.

Question 29 of 31 A 26-year-old male complains of double vision for two days. On examination you notice he has a partial ptosis to the left eye. The eye is depressed and abducted in the primary position. You perform an ocular motility examination in which the patient describes diplopia in all directions of gaze except in abduction of the left eye. This is confirmed with no limitation of movement on abduction to the left eye. What is the next most appropriate examination? (Please select 1 option) Colour vision Cranial nerve examination Fundal examination Pupillary examination Visual acuities

Pupillary examination This is the correct answer The symptoms and signs described here are consistent with that of a likely diagnosis of an isolated partial third nerve palsy. There is limitation of all directions of gaze except in abduction, as this is innovated by the abducens nerve. Examination suggests that there is partial ptosis, which indicates that the lesion may still be evolving. Pupillary examination will determine if the lesion is due to a compressive lesion which will disrupt the parasympathetic fibres on the surface of the third nerve. Possible causes include a posterior communicating artery aneurysm or tumour. Urgent neuroimaging is indicated. Colour vision assesses optic nerve function. Although cranial nerve examination is important in the conclusion of your full assessment of this patient (to assess whether other cranial nerves are involved) it does not assess the nature of the third nerve at this stage. Fundal examination assesses visualisation of the optic nerves. Visual acuities will not add any further information regarding the third nerve palsy.

Question 15 of 15 Core Questions A 40-year-old Caucasian male presented to the acute medical unit with bilateral strokes affecting the basal ganglia. Which of the following would support the diagnosis of stroke associated with mitochondrial disease? (Please select 1 option) Cardiac emboli Cerebral atherosclerosis Hypertension Sensorineural hearing loss Smoking

Sensorineural hearing loss This is the correct answer Stroke in a young person requires extensive investigation to exclude underlying causes, including: congenital heart defects metabolic abnormalities autoimmune disease, and genetic predisposition. Mitochondrial disease can present in mid-late adulthood as a stroke or stroke-like illness, that is, "ischaemic necrosis of brain tissue that occurs in the absence of any vascular occlusion". Typical features of stroke associated with mitochondrial disease include: young age (mid-40s) extra-neurologic features of mitochondrial disease such as diabetes and sensorineural hearing loss no source of cardiac emboli or cerebral atherosclerosis, and presence of a stroke in a region of the brain that does not correlate to vascular distributions (bilateral basal ganglia infarction). Reference: Mattman A, Sirrs S, Mezei MM. Mitochondrial disease clinical manifestations: An overview. BCMJ 2011;53:183-187.

Question 14 of 15 Core Questions A 37-year-old building contractor presents with shortness of breath, weight loss and cough with haemoptysis. He is a smoker and has a 20 pack-year history. He is Caucasian and there is no history of contact with TB or recent foreign travel. You are concerned that despite his age he is at risk of developing lung cancer. You commence him on antibiotics and request a CT thorax. As the advice offered to patients as part of health promotion is an essential component of patient management you advise him regarding downstream determinants. In public health, which of the following can be described as a "downstream" determinant? (Please select 1 option) Education Employment Local schools Smoking Working environment

Smoking This is the correct answer Patient education is a key part of patient management. As a physician it is important to understand which lifestyle factors can be highlighted for intervention. Determinants of health can be split into 'upstream', 'midstream' and 'downstream' determinants. Upstream determinants are those under national control. These include social, physical, economic and environmental factors such as education, employment, income and housing. Midstream determinants are intermediate level factors which affect populations such as local communities, schools or universities and include local availability of facilities for exercise and safe neighbourhoods. Finally, downstream determinants include individual lifestyle factors for example smoking, alcohol intake and diet. Reference: British Medical Association. Social Determinants of Health - What Doctors Can Do.

Question 10 of 31 A 38-year-old man is brought to hospital in status epilepticus. He is loaded with phenytoin intravenously and admitted to the high dependency unit for monitoring. CT head excludes an SOL or bleed. It transpires that he is not known to suffer with epilepsy but does have a past medical history of a traumatic subdural haematoma secondary to a road traffic accident seven years ago, which required a craniotomy. Which of the following antiepileptic drugs (AEDs) would you consider starting him on for maintenance therapy? (Please select 1 option) Carbamazepine Lamotrigine Gabapentin Phenytoin Sodium valproate

Sodium valproate This is the correct answer The risk of late-onset seizures increases over time. The underlying mechanism is similar for seizures developing post traumatic brain injury and post stroke, both of which may result in scar tissue formation. Post-stroke epilepsy is defined as two or more unprovoked seizures occurring after the acute phase of stroke. Choice of AED depends on a number of factors, including: the type of seizures (partial versus generalised) age tolerability to side effects, and polypharmacy etc. However, randomised controlled studies have shown superior efficacy of valproate in generalised seizures and lamotrigine in partial seizures. Reference: French JA. First-choice drug for newly diagnosed epilepsy. Lancet. 2007;369:970-1. Marson AG, Al-Kharusi AM, Alwaidh M, et al. The SANAD study of effectiveness of valproate, lamotrigine, or topiramate for generalised and unclassifiable epilepsy: an unblinded randomised controlled trial. Lancet. 2007;369:1016-26. Marson AG, Appleton R, Baker GA, et al. A randomised controlled trial examining the longer-term outcomes of standard versus new antiepileptic drugs. The SANAD trial. Health Technol Assess. 2007;11:iii-iv, ix-x, 1-134.

Question 3 of 31 A 78-year-old man who is independently mobile and lives with his son presents with collapse. Past history includes diet-controlled diabetes, hypertension and atrial fibrillation. This presentation is preceded by a lower respiratory tract infection for which his GP had commenced antibiotics. The patient was on day five of his treatment and was recovering from his symptoms when his legs gave way beneath him as he suddenly stood up. He was unable to get up thereafter and his son called for an ambulance. On examination the lower limbs are arreflexic with bilateral extensor plantars. A sensory level is detected at the umbilicus. Proprioception and vibration sense are intact. Power is 2/5 in the lower limbs and 5/5 in the upper limbs. Which of the following is the most likely diagnosis in this case? (Please select 1 option) Cauda equina syndrome Side effect of amoxicillin Spinal cord compression Spinal cord infarction Tranverse myelitis

Spinal cord infarction This is the correct answer A sudden onset of limb weakness and altered neurology in a person with atrial fibrillation is most likely to be due to embolic phenomena. Other differentials could potentially include tumour or bleed and an urgent MRI is indicated to rule out a compressive lesion. However, given the very sudden onset and a likely source of emboli spinal cord infarction is first on the list of differentials. Risk factors for vascular cord syndromes include: diabetes hypertension, and atrial fibrillation. All the other diagnoses on the list would present with neurological symptoms which progress over days to weeks. An adverse effect from amoxicillin use is unlikely to present this way.

A 64-year-old man presents with a two week history of worsening shortness of breath, lethargy and general malaise. Past medical history consists of a porcine aortic valve replacement four years previously for rheumatic heart disease, HTN, hypothyroidism and gout. On examination he is febrile at 38.2°C, BP 135/70 mmHg, pulse 120/min, oxygen saturations 98% on air. His JVP is not raised and he has no murmurs. Chest examination, abdominal and neurological examinations are unremarkable. There are no peripheral stigmata of infective endocarditis. His investigations are as follows: Hb 134 g/L (115-165) WCC 12.7 ×109/L (4-11) Platelets 399 ×109/L (150-400) CRP 35 (<5) Trop I <0.1 (<0.1) Serum sodium 137 mmol/L (137-144) Serum potassium 3.8 mmol/L (3.5-4.9) Serum urea 10.2 mmol/L (2.5-7.5) Serum creatinine 135 µmol/L (60-110) Chest x ray shows normal heart size and clear lungs. ECG shows sinus tachycardia 110 bpm with occasional ventricular ectopics. An echo was performed which showed a normal functioning aortic valve replacement with no para-valvular leaks. Nil evidence of any vegetations. Left ventricular function was normal with an ejection fraction of 65%. Urinalysis showed 3+ blood and 2+ protein only. Two separate blood cultures taken on admission grew Staphylococcus aureus. Using the above clinical information how many major/minor criteria does this patient fulfil of the modified Dukes's criteria for infective endocarditis? (Please select 1 option) Four minor One major, four minor One major, three minor One major, two minor Two major

The Dukes's diagnostic criteria, developed by Durack and colleagues in 1994, are generally used to make a definitive diagnosis of infective endocarditis (IE). As per the Dukes's criteria, IE is present if either two major, one major and three minor, or five minor criteria are present. Major criteria include: Two separate positive blood cultures with a typical organism consistent with infective endocarditis. e.g. Streptococcus viridans, Staphlococcus aureus, or the HACEK group of Gram negative bacteria (Haemophilus, Actinobacillus, Cardiobacterium hominis, Eikenella corrodens, Kingella) Two positive blood cultures (12 hours apart) with an organism consistent with infective endocarditis, and Positive echocardiogram showing oscillating intracardiac mass, abscess, new dehiscence of a prosthetic valve or new valvular regurgitation. Minor criteria include: Predisposing cardiac disease Recreational intravenous drug use Fever >38°C Vascular phenomena such as arterial emboli, pulmonary infarcts, splinter haemorrhages, splenomegaly or clubbing Immunological phenomena such as glomerulonephritis, Osler's nodes, Roth spots, and Positive blood culture (that does not meet a major criterion). In the above patient there is one major (two positive blood cultures of Staph. aureus) and three minor criteria (predisposing factor - tissue valve replacement, fever >38°C and immunologic phenomenon - glomerulonephritis with blood in the urine). Therefore he fulfils the criteria for infective endocarditis. Reference: MedCalc. Duke Criteria for Infective Endocarditis. Patient.info. Infective Endocarditis.

Question 12 of 15 You are asked to see a 66-year-old man on the high dependency unit with a history of collapse. He has a history of hypertension for which he takes lisinopril 10 mg. On admission his blood pressure was 80/50 mmHg and his temperature was 38°C. He has been taking some over-the-counter medications for hair loss, but he cannot recall their names. On examination he appears flushed, with a diffusely erythrodermic skin covering over 90% of his body. There are also a number of crusted lesions and scaling. He is complaining of itching. He has been admitted to HDU for monitoring, his normal medications stopped, with intravenous fluids and antihistamines administered. The dermatologist on-call feels this is erythroderma secondary to the hair-loss medications. A punch biopsy is planned for tomorrow and 'supportive treatment' suggested. Which of the following concerns you the most? (Please select 1 option) Possibility of underlying neoplasm Potential for HIV positive Risk of anaphylaxis Risk of dehydration Super added infection with methicillin-resistant Staphylococcus aureus (MRSA)

Super added infection with methicillin-resistant Staphylococcus aureus (MRSA) Correct This is a typical presentation of erythroderma which is a dermatological emergency and requires medical input for stabilisation of the patient and supportive treatment thereafter. Erythroderma is often idiopathic, but may be secondary to drugs, notably intravenous vancomycin. It may also be secondary to psoriasis, neoplasia and familial inflammatory conditions. The patient is at risk of dehydration but in this case intravenous fluids have been commenced and the patient is being closely monitored. However, the patient presented as haemodynamically unstable, itchy and pyrexial. Given his skin condition he is at high risk of developing super added infection with MRSA which may lead to sepsis. A very low threshold for cultures and early intervention with antibiotics is advocated in such cases. Erythroderma does not usually lead to anaphylaxis. HIV and neoplasia are potential underlying causes and should be investigated for in due course. DRESS (drug reaction with eosinophilia and systemic systems) may be the actual diagnosis in this case. Onset is within six weeks of taking the drug in question and resolution may take weeks to months.

Question 6 of 15 A 40-year-old car mechanic presents with symptoms of rapid, involuntary, arrhythmic, chaotic movements in his hands. They were first noticed by his wife, but he has presented now as they are becoming more pronounced in nature and he is unable to conduct his work as a mechanic. As a result of problems in his job, he has become depressed and his wife feels that he has become more aggressive in nature and more forgetful. Upon taking a history, there is a mention of the patient's father, who oddly also lost his job working as a mechanic due to poor performance (shaking of hands) and then committed suicide at age 55, before a diagnosis of any disease could be made. Assuming the son has now developed phenotypic features of the same disease process his father had, which of the following is correct? (Please select 1 option) The disease process is due to a mitochondrial disorder The genetics of the disease demonstrate anticipation The mode of inheritance is autosomal recessive The mode of inheritance is X linked The syndrome develops due to spontaneous mutation of the gene

The genetics of the disease demonstrate anticipation This is the correct answer Huntington's disease is an autosomal dominant neurodegenerative disorder. It occurs due to excessive triplet CAG repeats in the Huntington gene. The term anticipation is used to describe the mechanism of genetic mutation between successive generations. For Huntington's disease paternal inheritance can result in expansion of the repeat number of CAG repeats between successive generations, which causes an earlier and more severe phenotype. Huntington's disease (HD) is associated with the triad of chorea, psychiatric symptoms and dementia. There is a recognised risk of suicide in family members at risk of developing Huntington's disease. Reference: Collier R, Longmore M, Brinsden M. Oxford Handbook of Clinical Specialties. 7th edn. Oxford; Oxford University Press: 2006. p 646. Zhaurova, K. Genetic causes of adult-onset disorders. Nature Education. 2008;1:49.

A 71-year-old patient with known COPD phones the ambulance with chest pain and tightness. On their arrival the paramedics note he is cyanosed and commence oxygen via a reservoir bag. On arrival at hospital he is no longer cyanosed but noted to be drowsy. His arterial blood gas is as follows: FiO2 0.85 (estimated) pH 7.26 pO2 29.1 kPa pCO2 11.3 kPa The nursing staff recognise this as an indication for NIV and quickly leave him unattended to retrieve the NIV mask. What should you do during this time? (Please select 1 option) Drop to a 24% venturi mask and recheck the saturations Leave the current reservoir mask with 15 litres oxygen in place and await NIV Reduce the reservoir bag flow to 6 litres and recheck the saturations Remove the oxygen immediately, as it is contributing to the hypercapnia Switch to a slightly lower percentage venturi mask and recheck the saturations

The oxygen has taken away this patient's hypoxic drive, but sudden withdrawal of oxygen can also prompt rebound hypoxaemia, Rebound hypoxaemia is a major risk and may be more dangerous than the hypercapnic respiratory failure itself. Consequently, the current BTS oxygen guidelines recommend that oxygen therapy be stepped down gradually through sequential Venturi devices while monitoring saturation continuously. Sudden cessation of supplementary oxygen therapy can cause rebound hypoxaemia with a rapid fall in oxygen tension to below the tension that was present prior to the commencement of supplementary oxygen therapy, which could lead to respiratory arrest itself. Reference: O'Driscoll BR, Howard LS, Davison AG. BTS guideline for emergency oxygen use in adult patients. Thorax. 2008;63:vi1-68.

A 71-year-old patient with known COPD phones the ambulance for chest pain and tightness. On their arrival the paramedics note he is cyanosed and commence oxygen via a reservoir bag. On arrival at hospital he is no longer cyanosed but noted to be drowsy. His arterial blood gas is as follows: FiO2 0.85 (estimated) pH 7.26 pO2 29.1 kpa pCO2 11.3 kpa The nursing staff recognise this as an indication for NIV and quickly leave him unattended to retrieve the NIV mask. What should you do during this time? (Please select 1 option) Leave the current reservoir mask with 15 litres oxygen in place and await NIV Reduce the reservoir bag flow to 6 litres and recheck the saturations Remove the oxygen immediately, as it is contributing to the hypercapnia Switch to a 24% venturi mask and recheck the saturations Switch to a 40% venturi mask and recheck the saturations

The oxygen should be reduced in steps whilst monitoring the saturation aiming for the target of (usually) 88-92%. Sudden withdrawal of oxygen can cause a dangerous drop in the pO2. The oxygen has taken away this patient's hypoxic drive, but sudden withdrawal of oxygen can also prompt rebound hypoxaemia. Rebound hypoxaemia is a major risk and may be more dangerous than the hypercapnic respiratory failure itself. Consequently, the current BTS oxygen guidelines recommend that oxygen therapy be stepped down gradually through sequential venturi devices while monitoring saturation continuously. Sudden cessation of supplementary oxygen therapy can cause rebound hypoxaemia with a rapid fall in oxygen tension to below the tension that was present prior to the commencement of supplementary oxygen therapy, which could itself lead to respiratory arrest. Reference: O'Driscoll BR, Howard LS, Davison AG; British Thoracic Society. BTS guideline for emergency oxygen use in adult patients. Thorax. 2008;63:vi1-vi68.

A 27-year-old lady presents to you with shortness of breath and cough with clear sputum. She is ten days post partum. She underwent a caesarean section; both the patient and her baby are otherwise well. Her ECG is as follows: Which of the following investigations should be done urgently? (Please select 1 option) 24 hour tape Angiogram Bubble ECHO Echocardiogram (transthoracic) Myocardial perfusion scan

This lady is suffering with peri- or postpartum cardiomyopathy. Symptoms may present ten days before delivery and up to five months post delivery. The presentation is usually that of congestive cardiac failure. The echocardiogram would reveal a diastolic dysfunction. Treatment is as for heart failure. Other investigations to consider in such a presentation would be pulmonary embolus, hyperthyroidism and arrhythmia.

A 24-year-old lady presents to the acute medical unit with an exacerbation of her asthma. She is able to talk but struggles with full sentences. On examination you note she is thin and complains of breathlessness and a 'tight chest' for 24 hours. Air entry is equal bilaterally, with an expiratory wheeze throughout. Her oxygen saturation on air is 95%, respiratory rate is 35, pulse rate 117 bpm. An arterial blood gas (on air) is taken and reveals: pH 7.44 pCO2 5.9 kpa pO2 9.8 kpa HCO3 23.4 BE −1.52 A portable chest x ray reveals hyperexpanded lung fields with no pneumothorax or consolidation. What would be the next most appropriate management step? (Please select 1 option) Back-to-back inhaled beta agonist therapy Call ICU for further assessment Intercostal chest drain for pneumothorax IV aminophylline with loading dose of 5 mg/kg ideal body weight Start NIV/bi-level ventilation

The raised CO2 can imply the patient is becoming exhausted and may require assessment for intubation or monitoring on an intensive care unit. Asthma patients with rising CO2 should not be started on NIV as the problem is not related to hypoxic drive, but rather physical exhaustion. Reference: British Thoracic Society. Asthma Guideline.

A 64-year-old man is admitted to the Emergency department with a 100 ml upper GI bleed. He admits to indigestion which has increased over the past few days. He was recently discharged from the coronary care unit having received three stents following presentation with an NSTEMI. Medication includes aspirin, clopidogrel, ramipril, atorvastatin and bisoprolol. On examination his BP is 115/80 mmHg, pulse is 70 and regular. There is no postural drop in BP. His chest is clear and there are no signs of cardiac failure. There is mild tenderness in the epigastrium. Investigations reveal: Hb 110 g/L (135-180) WCC 9.3 ×109/L (4.5-10) PLT 180 ×109/L (150-450) Na 138 mmol/L (135-145) K 4.9 mmol/L (3.5-5.5) Cr 112 µmol/L (70-110) ECG shows Inferior Q waves, no other changes. CXR is clear. Upper GI endoscopy shows multiple duodenal erosions with bleeding on contact. Which of the following is most appropriate intervention with respect to the GI bleed? (Please select 1 option) Start omeprazole and continue antiplatelet agents Stop aspirin Stop aspirin and clopidogrel Stop clopidogrel Substitute dipyridamole for the clopidogrel

The risk of a further ischaemic cardiovascular event including an occlusive event involving one or more of the new stents is very high in the first few days after the procedure. As such the dual antiplatelet approach should be continued if possible as data suggest benefit up to at least one year post the stenting procedure. Given there are erosions only seen on endoscopy, and the patient is stable, starting PPI and continuing anti-platelet therapy is appropriate in this situation. Studies have shown that single agent antiplatelet therapy is inferior versus the dual approach. Stopping one agent is therefore not appropriate if it can be avoided. It is thought that some individuals may be resistant to one of aspirin or clopidogrel as an antiplatelet therapy; use of combination treatment overcomes this as a potential problem. Clopidogrel is also thought to be superior to dipyridamole in reducing the risk of myocardial infarction. It is clear that if further GI haemorrhage occurs then continuation of dual therapy may require review, but at this point continuing under PPI cover is the best approach. If a decision is taken to suspend use of one agent, clopidogrel as monotherapy is superior to aspirin.

Question 18 of 23 A 45-year-old man walks in to the Emergency department complaining of palpitations after going for a run. He denies chest pain, shortness of breath or dizziness. His blood pressure is 138/60 mmHg, however the heart rate is 198/min regular. He has been investigated by his GP for intermittent palpitations, mostly noted on exercise. His recent 24 tape has been normal. You attach him to an ECG monitor and the following ECG trace is noted: Which of the following would be the most appropriate line of management? (Please select 1 option) Beta blocker Catheter ablation for arrhythmogenic focus Defibrillation Digoxin Implantable cardioverter defibrillator (ICD)

The treatment decision for exercise-induced VT depends on the symptoms. If the symptoms are severe, such as presyncope or syncope, then catheter ablation of the focus is the treatment of choice. If symptoms are mild pharmacological therapy may be sufficient. This type of arrhythmia may spontaneously revert after an episode of exercise. However, if it does not it may degenerate into ventricular fibrillation and cause sudden cardiac death. Pharmacologic therapy improves symptoms in a significant proportion of patients. The initial choice for therapy is usually a beta blocker. Catheter ablation or implantation of an ICD are also options. If a patient is only mildly symptomatic (as in this case), a trial of medications would be warranted. Digoxin increases intracellular calcium and so can potentially trigger arrhythmogenic activity. Procainamide may prolong the QT interval.

Smoking is a lifestyle choice made by a significant percentage of the population. It is recognised as causing morbidity and mortality. Doctors are encouraged to educate their patients with regard to smoking cessation. Which of the following methods is the most effective way to stop smoking? (Please select 1 option) Bupropion (Zyban) as part of an NHS smoking cessation programme E-cigarettes one cartridge daily (~12 mg nicotine/cartridge) Hypnotherapy as part of an NHS smoking cessation programme Nicotine patches 21 mg/day No adjuncts

There are various forms of medication aimed at smoking cessation, the efficacy of which vary greatly depending on various studies. However, they are all advised as being equally effective when used in conjunction with an NHS programme. The least effective method is trying to stop with no help at all and has a success rate of no more than 3-10% at best. There is no evidence to suggest that hypnotherapy is any more effective than quitting without help. Champix and Zyban (trade names), in conjunction with support from the appropriate NHS professionals help increase this success rate to up to 20-25% effectiveness. Other methods of smoking cessation include patches, gum, and electronic cigarettes although the latter are still undergoing evaluation for NHS use. Reference: Barnes J, Dong CY, McRobbie H, et al. Hypnotherapy for smoking cessation. Cochrane Database Syst Rev. 2010;(10):CD001008. NHS Choices. Stopping smoking.

A 54-year-old South Korean man travels to the UK on business. He is quite unwell and is admitted to hospital with symptoms of a lower respiratory chest infection and associated pleuritic chest pain. Despite being treated with intravenous benzylpenicillin and clarithromycin as per local guidelines, he continues to deteriorate. You suspect this may be multidrug-resistant Streptococcus pneumoniae. Which of the following antibiotics are likely to help overcome resistance in this case? (Please select 1 option) Cefotaxime Ciprofloxacin Doxycycline Rifampicin Tazocin

There is an extremely high percentage of antibiotic resistance to Streptococcus pneumoniae in South Korea. Multidrug-resistant bacteria (aka 'superbugs') are a concern worldwide. However S. pneumoniae in South Korea has developed extensive drug resistance, and several cases of multidrug-resistant S. pneumoniae have resulted in mortality. Winter to early Spring is the period of greatest risk. Symptoms often begin with upper respiratory symptoms of acute onset which progresses into the lower respiratory tract. There is often associated pleuritic chest pain. Vancomycin and rifampicin can be added to treatment regimes to help to overcome resistance. By comparison up to 25% of S. pneumoniae are penicillin-resistant in the UK

An 81-year-old man with chronic obstructive pulmonary disease (COPD) is due to be discharged from the acute medical unit following an excerbation of COPD. On leaving he asks whether he can have oxygen at home for the times he feels breathless, as well as nebulisers, as they quickly make him feel better. His saturation on room air is 90%. His arterial gas shows the following FiO2 0.21 pH 7.41 pO2 7.2 kPa pCO2 5.1 kPa HCO3 24 What would be a reasonable management step? (Please select 1 option) Cancel admission and reassess as inpatient for oxygen requirements Discharge home with long term oxygen therapy (LTOT), and nebulisers review in six weeks Discharge home with long term oxygen therapy (LTOT), and respiratory review in 12 months Discharge home with nebulisers Discharge home with short burst oxygen therapy (SBOT) and nebulisers

There is no evidence that SBOT has any survival benefit or reduces admissions to hospital. Similarly many physicians advise if a patient is unwell enough to require nebulisers at home they ought to come in to hospital for assessment. In some patients nebulisers may improve quality of life and can be appropriate, but this needs discussion with their respiratory team or community medical team first. The only proven therapy for prolonging survival from the above treatments is LTOT. Patients with COPD should receive LTOT when their resting oxygen falls below 7.3 kPa (or under 8.0 kPa in the presence of polycythaemia, cor pulmonale or nocturnal hypoxaemia). However, the LTOT assessment should take place when the patient is stable, hence usually six weeks after discharge rather than whilst still in hospital, as they may simply be recovering from an acute illness. Once they meet the criteria they should be advised not to smoke in the presence of the oxygen, and to wear it for at least 15 hours a day (or longer) in order to obtain the benefits. Reference: British Thoracic Society. Home Oxygen Guideline- adults.

A 72-year-old man presents to the Emergency department with an episode of syncope whilst shopping at the local supermarket. According to a first responder he was gasping for breath just after the event, and it was difficult to feel a pulse. He has a history of hypertension which is managed by his GP with a combination of ramipril 5 mg and amlodipine 5 mg once daily. His BP is 162/110 mmHg, pulse is 85 and regular on arrival. He has a harsh ejection systolic murmur, and bilateral basal crackles on auscultation of his chest. Investigations reveal: Hb 138 g/L (135-180) WCC 7.3 ×109/L (4-10) PLT 200 ×109/L (130-400) Na 138 mmol/L (135-145) K 4.2 mmol/L (3.5-5.5) Cr 129 µmol/L (50-90) Troponin <0.05 - ECG shows LVH by voltage criteria. CXR shows increased cardiac size. ECHO reveals 55 mmHg gradient across the aortic valve. Which of the following is the strongest indication for surgery? (Please select 1 option) Age <80 Creatinine 129 µmol/L Gradient across the valve LVH Syncopal episode

Thinking about indications for valve replacement in aortic stenosis has changed over the past few years. Registries indicate that only 4% of sudden deaths occur in asymptomatic patients with aortic stenosis, and therefore taking into account operative mortality of 4-8% in patients >70 years, it is the appearance of symptoms which drives referral for surgery. Symptoms of course do not just include syncope, but also appearance of significant left ventricular failure. Both the creatinine of 129 µmol/L and the LVH may not solely be related to aortic stenosis, and may be due to co-existent atherosclerotic vascular disease. Whilst age <80 is associated with improved operative mortality vs. more elderly patients, the increased risk of sudden death associated with symptomatic AS is by far the biggest driver for surgical intervention.

A 38-year-old lady presents to the acute medical unit with a sharp pleuritic chest pain of sudden onset two days previously. She is normally fit and well with no previous history or family history of note. On examination her respiratory rate is raised at 22 breaths per minute. Her arterial blood gas reveals the following: FiO2 0.35 pH 7.44 pO2 9.8 kPa pCO2 3.9 kPa HCO3 24 Whilst awaiting a chest x ray her blood pressure drops to 66/30 mmHg. She becomes cyanosed but still conscious and complains of increasing heaviness on her chest. What is the next best management step? (Please select 1 option) 50 mg alteplase bolus Immediate initiation of IV heparin Immediate thrombolysis with streptokinase over two hours Low molecular weight heparin subcutaneous with urgent bedside echocardiography Urgent discussion for CTPA with outreach/ICU assistance

This case is strongly suggestive of a massive pulmonary embolism. The only difficulty here is that the diagnosis has not yet been confirmed by CTPA. However, trying to transfer the patient to CTPA whilst she is unstable is risky given the high mortality of massive PE. The correct management of a clinically suspected life threatening PE is alteplase 50 mg for immediate thrombolysis. If the blood pressure is stable but there are signs of right heart strain (for example, S1Q3T3 on ECG, troponin rise or right ventricle dilatation/interventricular straightening on echocardiography), then it is a suspected submassive PE. In this situation thrombolysis is not recommended, although individual cases should be discussed with respiratory specialists. Reference: British Thoracic Society. Pulmonary Embolism.

A 59-year-old painter and decorator comes to the acute medical unit complaining of a persistent cough, lethargy and increasing shortness of breath. He is a long term smoker. Over the past two months he has noticed a heavy, dull boring pain in his left chest and ribs. Examination reveals reduced air entry to the left mid hemithorax with saturations of 96% on air. He is able to talk in full sentences but quickly becomes breathless on mobilising short distances. The chest radiograph shows a moderate to large pleural effusion on the left hand side. What would be the best management step at this point? (Please select 1 option) CT thorax for possible malignancy Ultrasound guided diagnostic pleural tap Ultrasound guided diagnostic pleural tap, with repeat tap if first sample negative on cytology Ultrasound guided Seldinger chest drain and possible talc pleurodesis Ultrasound guided therapeutic pleural tap

This vignette is consistent with lung malignancy. However, malignancy cannot be confirmed without obtaining a specimen or seeing a suspected mass on imaging. A CT thorax at this stage may not be able to visualise the area under the fluid and so could miss a mass. A diagnostic tap may be helpful but does not help the patient. A chest drain will help relieve the breathlessness but if the fluid analysis returns negative further opportunities to investigate the pleural effusion will be lost. Pleural fluid can detect malignancy in 60% of cases. Sending a repeat sample does not tend to increase the yield and should be avoided. A therapeutic aspiration can remove approximately 1 litre of fluid to relieve breathlessness whilst keeping some of the effusion in the chest cavity. If the fluid analysis is not helpful, a thoracoscopy would then be indicated to investigate the symptoms. Once the fluid is removed a staging CT would be the next step. Reference: British Thoracic Society. Pleural Disease Guideline.

A 64-year-old man is about to have an elective cystoscopy for investigation of haematuria. He has undergone a bovine aortic valve replacement (AVR) eight years previously for rheumatic heart disease with no complications. Chest examination, abdominal and neurological examinations are unremarkable. His investigations are as follows: Hb 134 g/L (115-165) WCC 6.7 ×109/L (4-11) Platelets 399 ×109/L (150-400) CRP 3 (<5) Trop I <0.1 (<0.1) Serum sodium 137 mmol/L (137-144) Serum potassium 3.8 mmol/L (3.5-4.9) Serum urea 10.2 mmol/L (2.5-7.5) Serum creatinine 135 µmol/L (60-110) Drug history includes aspirin 75 mg, ramipril 2.5 mg, atorvastatin 40 mg, bisoprolol 2.5 mg. He has no drug allergies. What prophylaxis should he have against developing endocarditis? (Please select 1 option) Amoxicillin 1 g IV stat Gentamicin 3 mg/kg stat Gentamicin 80 mg stat None Tazocin 4.5 g IV stat

Though infective endocarditis (IE) is uncommon it is associated with serious sequelae including stroke, cardiac surgery or even death. In an effort to prevent this antibiotic prophylaxis has been given to at risk patients before interventional procedures for over 50 years. However, published data by the British Society for Antimicrobial Chemotherapy and the American Heart Association have shown lack of efficacy in preventing IE using various antibiotic prophylactic regimes. In 2008 NICE published clinical guidance on this topic for patients undergoing dental and certain non-dental interventional procedures. Patients at risk of IE include: those who previously had IE those who have had a valve replacement anyone who has valvular disease (either stenosis or regurgitation) anyone with hypertrophic cardiomyopathy those who have congenital heart disease (including surgically repaired conditions but excluding isolated atrial septal defect, repaired ventricular septal defect, repaired patent ductus arteriosus). Antibiotic prophylaxis is unnecessary in patients undergoing dental or non-dental procedures (including upper/lower GI endoscopy, cystoscopy, obstetric procedures including childbirth as well as bronchoscopy). However, any episodes of infection in at risk individuals should be investigated and treated promptly to prevent IE developing. Reference: NICE. Prophylaxis against infective endocarditis (CG64).

A 57-year-old man presents with palpitations which are fast and regular. The ECG strip confirms supra- ventricular tachycardia (SVT). He has a past medical history of asthma. You decide to treat him with intravenous verapamil. He reverts to sinus rhythm after a 5 mg dose. Three hours later you called to the ward to reassess the same man. Whilst examining him, you notice a bruise on his forehead. He mentions that he got up to go to the toilet whilst on the ward and woke up on the floor. This is all he remembers of the event. At this point he is back in SVT at a rate of 150 beats per minute, has a normal blood pressure, and has no chest pain, shortness of breath or dizziness. Which of the following is the most appropriate way of managing this patient? (Please select 1 option) Adenosine to diagnose underlying rhythm Amiodarone infusion Bisoprolol intravenously DC cardioversion Second dose of verapamil IV

Though this patient is haemodynamically stable when reassessed, he mentions a recent history of syncope, indicating that circulation has been compromised due to the fast heart rate. A second dose of verapamil would not be ideal in this situation. Given the history of asthma, both adenosine and beta blockers are contraindicated. Amiodarone is not a drug of choice for managing SVT. Treatment with elective DC cardioversion would be recommended.

Question 23 of 31 Core Questions A 48-year-old lady presents to you with a sudden severe headache in the occipital region. She has a history of migraines but this pain is different. She also has a history of her brother suffering an intracerebral haemorrhage. CT head is normal. The lumbar puncture results are too blood stained to interpret accurately. Given her history you request a cerebral angiogram. Based on the findings of this scan, what will your management be? (Please select 1 option) Commence treatment for meningitis Discharge on NSAIDS, follow up in headache clinic Repeat lumbar puncture for xanthochromia Transfer care to neurosurgery for intervention Trial of sumatriptan on the ward

Transfer care to neurosurgery for intervention This is the correct answer The cerebral angiogram image shows a highly tortuous basilar artery with a likely aneurysm. This lady suffered a subarachnoid haemorrhage, she was stabilised on fluids, nimodipine, analgesia and monitoring before having her aneurysm clipped at a tertiary neurosurgical unit.

Question 20 of 22 Core Questions A 70-year-old man presents with large amounts of melaena, he is shocked. His past medical history includes hypertension, mitral valve replacement (metallic valve), for which he is on warfarin and ischaemic heart disease. Investigations reveal: INR 2.3 Hb 70 g/L Platelets 100 ×109/L Observations show a heart rate of 110/min, sinus tachycardia, RR 23/min, BP 89/50 mmHg, sats 93% air. How would you balance the bleeding risk versus the risk of coagulation of the valve? (Please select 1 option) Transfuse the patient and continue on warfarin Transfuse the patient and reverse the warfarin using fresh frozen plasma (FFP) Transfuse the patient and send for immediate endoscopy Transfuse the patient and switch to intravenous unfractionated heparin Transfuse the patient and use prothrombin complex concentrate

Transfuse the patient and use prothrombin complex concentrate This is the correct answer In a case of warfarin versus bleeding where bleeding results in hypovolaemic shock, risk outweighs benefit and it is necessary to reverse the warfarin, even in a patient with a prosthetic valve. Risk of thrombotic complication in prosthetic valves per 100 patient years: 4.0 on no anticoagulation 2.2 on aspirin 1.0 on warfarin. Risk in the mitral valve is greater than in an aortic valve. Avoid the use of fresh frozen plasma (FFP) as there is no data to support the use of this products, and there are associated risks, such as thrombosis, volume overload, and transfusion reactions. An exception is the use of FFP to correct a coexisting coagulopathy (eg, from trauma). Reference: Cannegieter SC, Rosendaal FR, Briët E. Thromboembolic and bleeding complications in patients with mechanical heart valve prostheses. Circulation. 1994;89(2):635-41.

A 54-year-old ex-professional rugby player comes to the rapid access ambulatory clinic for review. He has been admitted twice to the Emergency department with episodes of collapse whilst out running during the past two months. Each time he reported an episode of very rapid palpitations. On the first occasion he did not actually lose consciousness. On the second occasion he thinks he did, and awoke to find a passer-by standing over him. The passer-by reported feeling a thready pulse with a very rapid rate. There is no significant past medical history. Examination in the clinic reveals a BP of 110/70 mmHg, pulse is 62 and regular. There are no murmurs and there are no signs of cardiac failure. Investigations reveal: Hb 140 g/L (135-180) WCC 7.0 ×109/L (4.5-10) PLT 202 ×109/L (150-450) Na 137 mmol/L (135-145) K 4.0 mmol/L (3.5-5.5) Cr 100 µmol/L (70-110) 12 lead ECG shows no significant abnormalities. CXR is normal. Which of the following is the most likely diagnosis underlying his collapses? (Please select 1 option) Brugada syndrome Congenital long QT syndrome Exercise-induced ventricular tachycardia Hypertrophic obstructive cardiomyopathy (HOCM) Paroxysmal atrial fibrillation

Transient exercise-induced VT is occasionally seen as a cause of collapse in fit sportsmen with a structurally normal heart. The normal examination in clinic, normal heart size, and lack of reported abnormalities on the resting ECG further supports this as the underlying diagnosis. In the first instance the priority is to assess systolic function and to rule out significant coronary artery disease. In the presence of normal systolic function and no underlying CAD, the condition is associated with a benign outlook. Beta blockade is considered as the first line intervention. Brugada syndrome is associated with characteristic ST changes and therefore cannot be the underlying cause of the palpitations seen here. QT prolongation would also have been identified on one of the many resting ECGs taken over the past few months. HOCM is associated with LVH. Paroxysmal AF is a more likely cause of presyncope rather than the collapse seen here.

Question 24 of 31 A 40-year-old female presents with blurry vision. Upon questioning she complains of headache upon Valsalva's manoeuvre, nausea and has vomited twice. She also describes tinnitus and diplopia. Ophthalmoscopy shows the following changes: What other visual symptom may she also describe? (Please select 1 option) Metamorphopsia Micropsia Nyctalopia Transient visual obscurations Tritanopia

Transient visual obscurations This is the correct answer Bilateral disc swelling with the associated symptoms must warrant urgent imaging to rule out a space-occupying lesion and associated raised intracranial pressure. Transient visual obscurations may also occur in cases of raised intracranial pressure when the vision momentarily disappears during spikes of intracranial pressure that usually occur during Valsalva's manoeuvre. Metamorphopsia is distortion in the vision. The patient usually complains of kinked and bending of straight edges. This symptom is present amongst patients with wet macular degeneration. Micropsia is when objects appear smaller than they really are. It is found amongst patients with macular pathology. Nyctalopia is night blindness, usually found in patients with retinitis pigmentosa or other congenital retinal degenerations. Tritanopia is yellow-blue colour blindness.

Question 14 of 22 A 45-year-old man is treated with propranolol and spironolactone for cirrhosis. He feels that he is passing smaller amounts of urine over the past few weeks despite taking 100 mg of spironolactone, and his ascites and peripheral oedema has been getting steadily worse. Examination reveals a BP of 105/70 mmHg, pulse is 80 and regular. He has marked ascites and peripheral pitting oedema of both lower limbs. Investigations show: Hb 105 g/L (135-180) WCC 7.2 ×109/L (4-10) PLT 122 ×109/L (130-400) Na 133 mmol/L (135-145) K 5.7 mmol/L (3.5-5.5) Cr 178 µmol/L (70-120) (122 some 5 weeks earlier) Urea 4.1 mmol/L (2.5-8) Albumin 24 g/L (35-50) Urine Blood + Protein + His spironolactone is withdrawn for two days with no apparent improvement in his renal function. Which of the following is the most appropriate next step? (Please select 1 option) IV 5% dextrose IV normal saline Therapeutic paracentesis Trial of terlipressin Withdraw the propranolol

Trial of terlipressin This is the correct answer The clinical picture here, with a gradual reduction in urine output, coupled with a rise in creatinine is consistent with hepatorenal syndrome. Management is somewhat difficult, with many of the options unlikely to impact on renal function. Terlipressin is worthy of a trial because it increases the blood pressure and potentially GFR by leading to splanchnic circulation vasoconstriction. Small pilot studies support its use1. Pushing IV fluids will only further exacerbate ascites or peripheral oedema. Therapeutic paracentesis may further compromise renal perfusion and therefore lead to a rise in creatinine. Withdrawing the propranolol will have some positive effect on blood pressure, but this may impact negatively on portal hypertension. As such a trial of terlipressin is the only potential option. Hepatorenal syndrome is a diagnosis of exclusion, the criteria for diagnosis are listed below2: Cirrhosis with ascites Serum creatinine >1.5 mg/dL (133 μmol/L) Absence of shock Absence of hypovolaemia as defined by no sustained improvement of renal function (creatinine decreasing to <133 μmol/L) following at least two days of diuretic withdrawal (if on diuretics), and volume expansion with albumin at 1 g/kg/day up to a maximum of 100 g/day No current or recent treatment with nephrotoxic drugs Absence of parenchymal renal disease as defined by proteinuria <0.5 g/day, no microhaematuria (<50 red cells/high powered field), and normal renal ultrasonography Reference: Solanki P, Chawla A, Garg R, et al. Beneficial effects of terlipressin in hepatorenal syndrome: a prospective, randomized placebo-controlled clinical trial. J Gastroenterol Hepatol. 2003;18:152-6. European Association for the Study of the Liver. EASL clinical practice guidelines on the management of ascites, spontaneous bacterial peritonitis, and hepatorenal syndrome in cirrhosis. J Hepatol. 2010;53:397-417.

Question 4 of 22 A 76-year-old man presents to you with symptoms of a viral gastroenteritis for 12 days. He is normally quite well and has just come back from a Caribbean cruise and is now suffering with loose motions. There is some blood rectally, though he is known to have haemorrhoids. He is able to tolerate oral diet, though has been off his food for a little while now. Given the history the GP had suspected this to be likely norovirus or rotavirus and prescribed simple paracetamol, loperamide and encouraged oral fluids. The patient is presenting to you now as the symptoms are still not settling. On examination the patient is haemodynamically stable and has a low grade fever. Abdomen is soft and digital examination confirms the presence of haemorrhoids and blood stained mucous. Blood results are as follows: Hb 111 g/L WCC 12 ×109/L Plts 512 ×109/L Na 136 mmol/L K 5.0 mmol/L Urea 7 mmol/L Creatinine 112 µmol/L Which of the following is correct? (Please select 1 option) Admit for blood and stool cultures and gastroenterology review Continue with paracetamol and loperamide, gastroenterology follow up Continue with paracetamol and loperamide, no need to admit Urgent colonoscopy as inpatient Urgent CT abdomen as outpatient

Urgent colonoscopy as inpatient This is the correct answer Reduced appetite, altered bowel motion, blood per rectally and anaemia in a patient over 65 years of age must be considered as 'red flag' symptoms and bowel cancer should be excluded. In the first instance treating for a viral gastroenteritis is probably reasonable, particularly given the history of being aboard a cruise ship where norovirus and rota virus are notoriously commonplace. However, a viral gastroenteritis is usually self limiting taking between seven to 10 days. This patient's symptoms have continued beyond that time frame and there are red flags present, including anaemia in a male patient. In this case given the treatment options available, It would be pertinent to conduct a colonoscopy urgently to exclude malignancy before discharging to follow up.

Question 16 of 22 A 52-year-old man presents with right upper quadrant pain and raised blood sugars. His back ground history includes type 2 diabetes, alcoholic liver disease, smoker. On examination the patient looks icteric, abdomen is soft but tender in the right upper quadrant, the patient is pyrexial. Blood results are as follows: Hb 122 g/L WCC 25.3 ×109/L Plts 489 ×109/L U&E normal Bilirubin 110 µmol/L Alk phosphatase 230 IU/L ALT 900 IU/L Albumin 33 g/L Glucose 18 mmol/L In order to confirm the underlying diagnosis, which of the following would be the most appropriate first line radiological investigation? (Please select 1 option) CT abdomen CT KUB ERCP MRI abdomen US abdomen

US abdomen Correct The triad of fever, jaundice and right upper quadrant pain points to a diagnosis of cholangitis. This is commonly referred to as "Charcot's triad". Cholangitis is superadded infection in the bile duct, usually due to stones but may be secondary to stricture, tumour, a pre-existing stent or sludge. Ultrasound can distinguish between cholangitis and cholecystitis which appears differently on ultrasound. Also ultrasound is often more accessible as a first line investigation than a CT. Gold standard for imaging the biliary tree would be MRCP, however it is more difficult to schedule and would not be considered a first line investigation in most healthcare facilities. Treatment includes blood cultures to identify the offending organism and aggressive management with antibiotics and fluids. ERCP (which is not a radiological investigation, it is an intervention) may be required to remove the duct obstruction.

Question 30 of 31 A 57-year-old woman is admitted with a three day history of headache, dizziness (world spinning around her) and vomiting. Symptoms are worse on first sitting up in the morning. She first noticed her symptoms after a vigorous episode of sneezing. She informs you that she has been suffering with a viral illness. Her past medical history includes hypertension, previous breast cancer (now discharged) and smoker of 30 cigarettes per day. On examination, she is anxious, there are no focal neurological signs, the gait is unsteady but the patient is able to ambulate. A CT scan of head is normal. What is the most likely diagnosis? (Please select 1 option) Cerebellar haemorrhage Space occupying lesion Vertebral artery dissection Vertebrobasilar insufficiency Vestibular neuritis

Vestibular neuritis Correct Vestibular neuritis is characterised by the rapid onset of severe, persistent vertigo, nausea, vomiting, and gait instability. When this is combined with unilateral hearing loss the syndrome is known as labyrinthitis. Vertigo is a common presentation to the acute medical unit. Any of the above diagnoses would fit the story with which this lady presented. However in the absence of focal neurological and with a normal CT head the most likely diagnosis is vestibular neuritis.

Question 11 of 31 A 46-year-old lady presents with dizziness, headache and nausea. You conduct a head impulse test and find a positive vestibular ocular reflex (VOR). Which of the following are likely to be the diagnosis and a result of this bedside test? (Please select 1 option) Benign paroxysmal positional vertigo Cerebellar stroke Cerebellar tumour Meniere's disease Vestibular neuritis

Vestibular neuritis Correct Vestibular neuritis is inflammation of the vestibular nerve resulting in a sudden onset of dizziness, nausea and vomiting (mild headache may be present as a constitutional symptom). Diagnosis is usually made clinically as blood tests are not helpful. An MRI may image the lesion. On examination the vestibular ocular reflex is positive and nystagmus is attenuated by fixation, which is unidirectional with the fast phase away from the affected ear. Labyrinthitis, although on a similar spectrum to vestibular neuritis, is a different diagnosis. It incorporates the peripheral sensory organs for balance and hearing, Symptoms of labyrinthitis occur when there is inflammation of the membranous labyrinth. Patient presentation is very similar to vestibular neuritis. The head impulse test is helpful when differentiating vestibular neuritis from stroke. The patient is asked to fix their gaze on a point and the head is rapidly rotated. If the eyes do a corrective shift once the head stops moving rather than maintaining contact throughout the movement, this is deemed suggestive of a vestibular disorder. Reference: Patient.info. Vestibular Neuritis.

Question 6 of 31 Core Questions A 23-year-old male presents with sudden loss of vision to his left eye. He is a type 1 diabetic with evidence of microalbuminuria. He describes his vision as having "lots of floaters". His vision in this eye is counting fingers. There is no relative afferent pupillary defect (RAPD) in either eye. You attempt to examine his eye with an ophthalmoscope and notice an absent red reflex in the affected eye. What is your diagnosis for the left eye? (Please select 1 option) Cataract Diabetic papillopathy Exudative diabetic maculopathy Rubeosis iridis Vitreous haemorrhage

Vitreous haemorrhage Correct Symptoms and signs are consistent with that of a vitreous haemorrhage. Vitreous haemorrahage is likely to be caused by neovascularisation due to proliferative diabetic retinopathy (PDR). There is an increased risk of PDR in patients with renal impairment and type I DM. Cataract visual loss is gradual as opposed to sudden in this case. Also, floaters are not consistent with cataract. Visual loss in diabetic papillopathy is also gradual. In addition RAPD will be present along with a normal red reflex. Floaters and absent red reflex are not consistent with maculopathy.

Question 26 of 31 An 84-year-old female presents with distorted vision for two weeks to her right eye only. Her left eye is has evidence of drusen in the macular area. Upon ophthalmoscopy you notice the following changes below: What would be your diagnosis and likely treatment for this lady? (Please select 1 option) Diabetic maculopathy and laser treatment Diabetic maculopathy and steroid implants Dry macular degeneration and dietary advice Wet macular degeneration and intravitreal anti-VEGF therapy Wet macular degeneration and PDT laser

Wet macular degeneration and intravitreal anti-VEGF therapy This is the correct answer The fundal picture shows changes consistent with that of wet macular degeneration. Features to support this include haemorrhage and drusen, which are the yellow lesions adjacent to the haemorrhage. This patient needs to be referred to the HES. NICE approved treatment for this condition, providing the vision is within the criteria, is monthly intravitreal anti-VEGF therapy. Haemorrhage is not consistent with diabetic-related eye disease. There is no evidence of diabetic retinopathy in the affected eye and there are no changes in the opposing eye. Dry macular changes would only consist of the drusen; there would be no evidence of macular oedema or haemorrhage. PDT laser was the treatment for wet ARMD pre-anti-VEGF.

A 87-year-old patient with COPD presents with increasing drowsiness and desaturation on air after being treated in the community by his general practitioner for a suspected right lower lobe pneumonia. He is able to talk but is physically quite frail. An arterial blood gas reveals the following (taken on 28% venturi mask): pH 7.27 pO2 8.05 kpa pCO2 9.16 kpa HCO3 29.1 BE −2.7 What would be a reasonable next step? (Please select 1 option) Immediate intubation and transfer to ICU Initiation of CPAP with arterial blood sample after 30 minutes Initiation of non-invasive continuous positive pressure ventilation (CPAP) with initial setting of 5 cmH2O Initiation of non-invasive ventilation with initial setting of 10/5 Initiation of non-invasive ventilation with initial setting of 20/5

When starting non-invasive ventilation (NIV) initial settings of 10/5 should be started as they are well-tolerated. A setting of 20/5 may be achieved after increasing the pressure in increments of 2-5 cm over 20 minutes. It is unlikely that this man will benefit from intubation and ICU given his frailty, advancing age and irreversible lung disease. His problem is CO2 retention so CPAP alone will help oxygenation but not address the carbon dioxide excretion. NIV or BIPAP will help here, and should be started on a low pressure and built up quickly. This allows the conscious and alert patient to adjust to the mask before increasing the pressure, which should be done over the next few minutes. Reference: Royal College of Physicians. Non-invasive ventilation in chronic obstructive pulmonary disease.


Conjuntos de estudio relacionados

Chapter 12 Concerns of Green Construction

View Set

Chapter 19: Complementary and Alternative Medicine

View Set

Chapter 15 SMost of the elements in the periodic table can be described as: (metals, nonmetals, metalloids)cience

View Set

N436 Inflammation, Wound Healing, SCI, Burn Q's

View Set

test 3 practice questions from pp

View Set